Sunteți pe pagina 1din 31

Test

1. vuelta Distancia

Psiquiatra

Test

1. vuelta Distancia

Psiquiatra
1)
2)
3)
4)
5)

TRASTORNOS NEURTICOS
1.

Una paciente presenta desde su adolescencia episodios


recurrentes de palpitaciones y mareo con sensacin de prdida
de estabilidad; ocasionalmente ha sufrido desvanecimientos;
tras repetidas exploraciones somticas negativas comienza
tratamiento psiquitrico. Cul de los siguientes cuadros es
RARO que complique el trastorno que sufre la paciente?

6.

En la etiologa del trastorno obsesivo NO es cierto:


1)

1)
2)
3)
4)
5)

Abuso de sedantes.
Trastorno obsesivo.
Depresin.
Alcoholismo.
Ansiedad crnica.

2)
3)
4)

2.

Ante un paciente que est en plena crisis de angustia, una de


las siguientes medidas es INEFICAZ:
1)
2)
3)
4)
5)

3.

5)

7.

Cul de las situaciones que se mencionan es MENOS probable


que desencadene miedo en un agorafbico?
1)
2)
3)
4)
5)

4.

Buspirona.
Lorazepam sublingual.
Control de la respiracin.
Diazepam oral.
Explicacin de la benignidad del cuadro.

Metro.
Centros comerciales.
Hablar en pblico.
Ascensores.
Aglomeraciones.

Psicoanlisis.
Sertralina.
Fenelcina.
Lorazepam.
Clomipramina.

Uno de los siguientes sntomas NO forma parte de los criterios


diagnsticos del trastorno por estrs postraumtico:

La fobia social es ms DIFCIL de diferenciar de:


1)
2)
3)
4)
5)

1)
2)
3)
4)
5)

Trastorno esquizoide.
Distimia.
Personalidad evitativa.
Agorafobia.
Esquizofrenia residual.
9.

5.

Existe una asociacin fuerte entre el trastorno de la Tourette


y el TOC.
Algunos casos infantiles se relacionan con infecciones
estreptoccicas.
Hay una disfuncin de los circuitos corticosubcorticales
frontales.
El elemento psicopatolgico ms tpico es la sensacin de
duda.
Se ha demostrado una alteracin selectiva de la neurotransmisin noradrenrgica.

Cul de las opciones de tratamiento parece la ms potente


para un paciente dominado por la necesidad de comprobar
cada pequeo acto cotidiano un nmero jo de veces, hecho
ste que le provoca gran ansiedad, pero que dice ser incapaz
de controlar?
1)
2)
3)
4)
5)

8.

La contaminacin.
La duda.
La simetra.
La soledad.
La enfermedad.

NO es frecuente que los pacientes obsesivos estn preocupados


por:

CTO Medicina C/Francisco Silvela, 106 28002 - Madrid

Pesadillas recurrentes con el suceso traumtico.


Amnesia de detalles importantes del acontecimiento.
Estado de nimo depresivo.
Respuesta de sobresalto exagerada.
Evitacin de los lugares relacionados con el suceso.

La presencia de mltiples sntomas fsicos para los que no se


encuentra una causa evidente dene al trastorno conocido
como:

Tfno. (0034) 91 782 43 30/33/34 E-mail: secretaria@ctomedicina.com www. ctomedicina.com

Test

1. vuelta Distancia

Psiquiatra
1)
2)
3)
4)
5)
10.

Amnesia.
Fuga.
Personalidad mltiple.
Despersonalizacin.
Convulsiones.

17.

Activando el receptor serotoninrgico 5HT1a.


Modicando la permeabilidad neuronal al calcio.
Potenciando la funcin GABArgica.
Interriendo con los sistemas de segundo mensajero.
Bloqueando la recaptacin noradrenrgica.

18.
1)
2)
3)
4)
5)
13.

Naloxona.
Flumacenil.
N-acetilcistena.
Trihexifenidilo.
Fisostigmina.

19.
1)
2)
3)
4)
5)
14.

Diazepam.
Cloracepato dipotsico.
Clordiacepxido.
Alprazolam.
Lorazepam.

1)
2)
3)
4)
5)

Midazolam.
Flurazepam.
Lorazepam.
Alprazolam.
Clonazepam.

20.

TRASTORNOS AFECTIVOS. FRMACOS ANTIDEPRESIVOS


Y ESTABILIZADORES DEL HUMOR
15.

La forma ms frecuente de trastorno del estado de nimo es,


de entre las siguientes:
1)
2)
3)
4)

Enfermedad bipolar.
Distimia.
Trastorno ciclotmico.
Depresin mayor.

CTO Medicina C/Francisco Silvela, 106 28002 - Madrid

Predominio de los sntomas somticos.


Quejas cognitivas.
Alto riesgo de suicidio.
Frecuente ideacin delirante.
Mala respuesta al electrochoque.

Acude a su consulta una mujer de 21 aos que lleva ms de dos


aos y medio sintindose cansada, con dicultad para concentrarse en los estudios. Su familia reere que, durante este tiempo,
tambin ha engordado unos 7 kilos debido a que come ms de
lo que lo haca antes. En esta paciente, usted espera encontrar
con ms probabilidad:
1)
2)
3)
4)
5)

Cul de las siguientes benzodiacepinas tiene una vida media


menor?

Depresin mayor.
Trastorno mixto ansioso-depresivo.
Personalidad depresiva.
Neurastenia.
Distimia atpica.

Dentro de los cuadros depresivos del anciano, NO es frecuente


que encontremos:
1)
2)
3)
4)
5)

En un paciente con hepatopata preferiremos utilizar una de las


siguientes benzodiacepinas:

Depresin melanclica.
Depresin psictica.
Depresin atpica.
Depresin neurtica.
Depresin puerperal.

Seale el trastorno afectivo que sufre una paciente de 35 aos


que desde su adolescencia experimenta de forma casi constante sntomas depresivos de intensidad leve, asociados con
una sensacin de cansancio enorme y una clara tendencia al
sueo, y cuyos sntomas apenas desaparecen unas semanas al
ao, empeorando de forma dramtica cuando la paciente sufre
un problema sentimental:
1)
2)
3)
4)
5)

Ante un paciente en el que sospechamos una intoxicacin por


alprazolam, el tratamiento de eleccin es:

Melancola.

El concepto de depresin mayor incluye en la DSM-IV diversos


subtipos de depresiones, entre los que NO se incluye:
1)
2)
3)
4)
5)

Las benzodiacepinas ejercen su accin:


1)
2)
3)
4)
5)

12.

16.

En la clasicacin DSM, NO se incluye dentro de los trastornos


disociativos:
1)
2)
3)
4)
5)

11.

5)

Dismorfofobia.
Hipocondra.
Sndrome de Briquet.
Conversin histrica.
Trastorno facticio.

Insomnio de conciliacin
Despertar precoz
Somnolencia excesiva diurna.
Despertares nocturnos frecuentes.
Sueo no reparador.

Una mujer de mediana edad, sin antecedentes psiquitricos


de inters, que presenta una profunda tristeza desde hace 6
meses as como incapacidad de disfrutar de cosas que antes le
hacan pasrselo bien. Su marido cuenta que ya nada parece
hacerle gracia, que come menos y que ha perdido 6 kilos en los
ltimos 3 meses. En la anamnesis, usted esperara encontrar que
la paciente mejora:
1)
2)
3)
4)
5)

Por la maana.
Con los IMAO ms que con tricclicos.
En relacin con factores externos.
Al terminar el da.
En verano.

Tfno. (0034) 91 782 43 30/33/34 E-mail: secretaria@ctomedicina.com www. ctomedicina.com

Test

1. vuelta Distancia

Psiquiatra
21.

La presencia de un sndrome depresivo leve y autolimitado tras


la prdida de un ser querido recibe el nombre de:
1)
2)
3)
4)
5)

22.

27.

2)
3)

Trastorno distmico.
Depresin farmacgena.
Depresin psictica.
Tristeza postparto.
Esquizofrenia catatnica.

4)
5)

Un paciente de 62 aos lleva un mes prcticamente sin realizar


ninguna actividad, no tiene ganas ms que de estar en la cama y
dice no tener fuerzas ni para acabar con su vida. Asimismo, est
convencido de estar arruinado a pesar de que, segn su familia,
mantienen una economa desahogada. El personal de enfermera
ha observado que est algo ms activo por la noche y que coge
bien el sueo, pero se despierta a las 3 de la madrugada. Tras
recibir tratamiento antidepresivo con imipramina empieza a
estar ms inquieto e irritable, hablar durante horas seguidas y
con tendencia a la procacidad y a la desinhibicin sexual. Qu
debemos considerar en primer lugar?
1)
2)
3)
4)
5)

24.

30.

Inicio a una edad ms tarda en el depresivo.


Ms antecedentes familiares afectivos en el bipolar.
Duracin ms larga de las recadas en el depresivo.
Alto riesgo de suicidio en el bipolar.
Equilibrio entre sexos en el depresivo.

NO sugiere bipolaridad en un paciente depresivo:


1)
2)
3)
4)
5)

Debut en el puerperio.
Antecedentes familiares de mana.
Presencia de ideacin delirante incongruente.
Inicio en la adolescencia.
Provocacin de hipomanas con antidepresivos.

CTO Medicina C/Francisco Silvela, 106 28002 - Madrid

Trastorno por angustia.


Narcolepsia-cataplejia.
Dolor crnico neuroptico.
Cefalea tensional.
Abstinencia alcohlica.

A la hora de seleccionar un antidepresivo para el tratamiento


de un paciente, cul termina siendo el principal factor implicado?
1)
2)
3)
4)
5)

31.

Propranolol.
Isoniacida.
Neurolpticos.
Reserpina.
Anticonceptivos hormonales.

En una de las siguientes indicaciones NO se utilizan antidepresivos:


1)
2)
3)
4)
5)

Dentro de las diferencias entre el trastorno bipolar y la depresin


recurrente, NO encontramos:
1)
2)
3)
4)
5)

25.

Que presenta una reaccin psicolgicamente normal tras


la recuperacin de la depresin.
Que padece un trastorno bipolar, desconocido hasta ese
momento.
Que el tratamiento antidepresivo est induciendo un cuadro
maniforme.
Que estn apareciendo rasgos patolgicos de personalidad,
ocultos por la depresin.
Que est simulando, pues esos sntomas no son congruentes
en un trastorno afectivo.

Varn, 25 aos, con distimia de inicio precoz, sin antecedentes


de suicidio.
Mujer, 46 aos, con depresin endgena unipolar.
Mujer, 85 aos, con depresin reactiva tras su reciente
mudanza.
Varn, 72 aos, con melancola delirante.
Mujer, 19 aos, con depresin bipolar y antecedentes
familiares de suicidio.

Las listas de frmacos capaces de causar depresin son muy


largas; sin embargo, uno de los siguientes NO se asocia con
depresin sino con mana:
1)
2)
3)
4)
5)

29.

Depresin mayor y mana.


Depresin mayor sobre una distimia.
Depresin mayor e hipomana.
Mana unipolar (sin depresin asociada).
Depresin invernal y episodios hipertmicos en verano.

Cul de los siguientes pacientes presenta un mayor riesgo de


suicidio?
1)

28.
23.

Se llama trastorno bipolar-2 a la siguiente combinacin de


sndromes afectivos:
1)
2)
3)
4)
5)

Melancola.
Depresin reactiva.
Duelo no complicado.
Depresin mayor.
Neurosis depresiva.

D su juicio diagnstico en el caso de una mujer de mediana


edad y sin antecedentes psiquitricos de inters, que presenta
profunda tristeza y llanto, estando convencida de padecer una
enfermedad mortal en castigo por los pecados que ha cometido.
Ha dejado de tomar alimento alguno, est postrada en la cama
y a duras penas responde a las preguntas que se le hacen:
1)
2)
3)
4)
5)

26.

Los antecedentes familiares de respuesta a un frmaco.


El subtipo sintomtico de depresin.
El antecedente de una respuesta personal al frmaco.
El perl potencial de efectos adversos.
Las preferencias del paciente.

Una vez superada la fase aguda de una depresin, se recomienda:


1)
2)
3)
4)
5)

Retirar el tratamiento antidepresivo.


Reducir la dosis a la mitad y mantener el tratamiento 6
semanas ms.
Mantener al menos 6 meses la misma dosis que produjo la
mejora.
Dejar una dosis mnima y ecaz para prevenir recadas de
forma indenida.
Comentar al paciente que tome el frmaco segn se encuentre.

Tfno. (0034) 91 782 43 30/33/34 E-mail: secretaria@ctomedicina.com www. ctomedicina.com

Test

1. vuelta Distancia

Psiquiatra
32.

Los antidepresivos tricclicos renen las siguientes caractersticas, EXCEPTO:


1)
2)
3)
4)
5)

33.

3)
4)
5)

2)
3)
4)
5)

Los IMAO reversibles (RIMA) han resultado poco ecaces.


Los IMAO clsicos precisan de una dieta para evitar crisis
serotoninrgicas.
En la depresin atpica los IMAO obtienen excelentes resultados.
Los IMAO selectivos de la MAO-B se usan en la enfermedad
de Parkinson.
Est contraindicada su combinacin con ISRS.

Se puede emplear como primera opcin teraputica en las


depresiones con alto riesgo suicida.
Sus escasas complicaciones permiten utilizarlo con seguridad en embarazadas.
A da de hoy est formalmente contraindicada en cualquier
forma de esquizofrenia.
Requiere aplicar al paciente anestesia general de corta
duracin y miorrelajacin.
La hipertensin intracraneal es contraindicacin de este
tipo de terapias.

2)
3)
4)
5)
40.

4)

CTO Medicina C/Francisco Silvela, 106 28002 - Madrid

Uno de los siguientes datos acerca de la epidemiologa de la


esquizofrenia es FALSO:
1)
2)
3)

Niveles inferiores a 0,4 mEq/l son, en general, inecaces.


Niveles superiores a 1,5 mEq/l se asocian con toxicidad.
Durante el perodo de mantenimiento suele bastar con
litemias inferiores a 1,0 mEq/l.
En las fases agudas manacas se recomiendan litemias
superiores a 1,0 mEq/l.

Mana disfrica.
Recadas estacionales.
Mana secundaria.
Ciclacin rpida.
Fase bipolar mixta.

TRASTORNOS PSICTICOS. FRMACOS ANTIPSICTICOS

Una de las siguientes armaciones acerca de las cifras de litemia


es FALSA:
1)
2)
3)

Suspender el litio e instaurar el topiramato como base de


su tratamiento.
Mantener el litio y aadir carbamacepina o valproico.
Quitar el litio y aadir el valproico y un ISRS, pues la paciente
est deprimida.
Mantener el litio y aadir olanzapina.
Quitar el litio y realizar terapia de mantenimiento con TEC.

Uno de los siguientes pacientes NO presenta una mala respuesta al litio:


1)
2)
3)
4)
5)

41.

Test de embarazo.
Sodio y potasio.
Electrocardiograma.
Electroencefalograma.
Creatinina y urea.

Tratamiento concomitante con carbamacepina.


Uso de nifedipino como antihipertensivo.
Sobreingesta hdrica.
Toma de indometacina por un esguince.
Prescripcin de teolina para una enfermedad pulmonar
obstructiva crnica.

Una paciente de 43 aos, diagnosticada desde los 22 aos de


trastorno bipolar y sin recadas hasta el ltimo ao tomando
exclusivamente litio, durante este ao ha presentado 5 episodios afectivos que su psiquiatra relaciona con la instauracin
de paroxetina ante fases depresivas leves y nalmente ha diagnosticado a la paciente de cicladora rpida. En estos momentos
est ligeramente deprimida. Cul de las siguientes opciones
teraputicas le parece ms adecuada?
1)

A la hora de iniciar un tratamiento con litio, una de las pruebas


siguientes resulta INNECESARIA:
1)
2)
3)
4)
5)

37.

39.

Acerca de la terapia electro-convulsiva (TEC), seale lo FALSO:


1)

36.

Disfunciones sexuales.
Somnolencia.
Estreimiento.
Poliuria.
Sequedad de piel y mucosas.

En las fases agudas depresivas se recomiendan litemias


inferiores a 1,0 mEq/l.

Un paciente bipolar en tratamiento con litio ingresa en Urgencias


por un sndrome confusional; al determinar la litemia se obtiene
una cifra de 3,1 mEq/l, cuando haca una semana su litemia era
de 1,0 mEq/l; suponiendo que el paciente no haya variado la
dosis prescrita, cul de las siguientes circunstancias explicara
esta cifra?
1)
2)
3)
4)
5)

NO es cierto sobre los frmacos llamados IMAO:


1)
2)

35.

38.

Los ISRS son los antidepresivos ms usados en la actualidad, entre


otras razones porque no producen apenas efectos secundarios
graves; sin embargo, muchos pacientes se van a quejar de:
1)
2)
3)
4)
5)

34.

Cardiotoxicidad.
Bajo precio.
Importantes efectos anticolinrgicos.
Alta letalidad en sobredosis.
Inicio del tratamiento a dosis teraputicas.

5)

4)
5)
42.

La prevalencia-vida es del 1%.


En varones se describe un comienzo ms precoz.
Hay una mayor incidencia de enfermedad en clases sociales
bajas.
Existe un exceso de esquizofrnicos que han nacido en los
meses fros.
La esquizofrenia duplica el riesgo de muerte.

El concepto actual de esquizofrenia se apoya en los siguientes


datos SALVO:
1)

Curso habitualmente crnico y deteriorante.

Tfno. (0034) 91 782 43 30/33/34 E-mail: secretaria@ctomedicina.com www. ctomedicina.com

Test

1. vuelta Distancia

Psiquiatra
2)
3)
4)
5)
43.

Cul de los siguientes factores NO se incluye dentro de las


causas de vulnerabilidad a la esquizofrenia?
1)
2)
3)
4)
5)

44.

48.

50.

Bloqueo del pensamiento.


Descarrilamiento.
Pensamiento disgregado.
Percepcin delirante.
Idea delirante secundaria.

El debut precoz de la esquizofrenia se asocia con un peor pronstico de la enfermedad, como todos los datos siguientes,
EXCEPTO uno:

3)

1)
2)
3)
4)
5)

5)

La probabilidad de recada en un paciente esquizofrnico NO


est en relacin con:

CTO Medicina C/Francisco Silvela, 106 28002 - Madrid

4)

53.

Bloqueo serotoninrgico 5HT1a.


Estmulo GABArgico.
Inhibicin de la COMT.
Antidopaminrgico D2.
Anticolinrgico muscarnico.

Una de las siguientes armaciones sobre los antipsicticos


tpicos es FALSA:
1)
2)

Sexo masculino.
Antecedentes familiares de depresin.
Personalidad previa paranoide.
Escasa productividad psictica.
Ausencia de desencadenantes.

Formacin reactiva.
Anulacin.
Interpretacin delirante.
Aislamiento del afecto.
Fantasa.

Cul es el mecanismo de accin principal de los antipsicticos


clsicos?
1)
2)
3)
4)
5)

52.

Descarrilamientos.
Percepciones delirantes.
Aplanamiento afectivo.
Pseudoalucinaciones auditivas.
Conducta catatnica.

Suele comenzar en edad juvenil.


No produce deterioro del paciente.
Tiene tendencia a la cronicidad.
Podra presentar delirios de perjuicio.
Se acompaa de alucinaciones

Un compaero de tu trabajo comienza a sospechar de la delidad


de su novia, quien trabaja en otro departamento de la empresa,
cercano al tuyo; cada vez que coincids en la cafetera y os ve
hablando, tiene la certeza de que estaris quedando para ms
adelante y si no hablis cree que se debe a que os habis sentido
vigilados, cmo se denomina este fenmeno psicopatolgico?
1)
2)
3)
4)
5)

51.

Eco del pensamiento.


Alucinaciones visuales.
Robo del pensamiento.
Vivencias de inuencia.
Delirios de control.

Su conciencia de enfermedad.
Los efectos secundarios del tratamiento.
El tipo de esquizofrenia.
El cumplimiento de la medicacin.
La emocin expresada en la familia.

Una paciente le reere que desde hace aos un famoso cantante


le maniesta su amor a travs de insinuaciones o gestos en sus
intervenciones pblicas. Ha tratado, sin xito, de comunicarse
con l mediante llamadas telefnicas, cartas e incluso yendo a
su domicilio, por lo que fue denunciada. No sufre alucinaciones
y su capacidad de juicio, fuera del tema citado, es totalmente
adecuada. Para distinguir este cuadro de una esquizofrenia,
usted se apoyara en que la enfermedad de la paciente:
1)
2)
3)
4)
5)

Cul de los sntomas siguientes aparece con mayor claridad en


la fase residual de la esquizofrenia?
1)
2)
3)
4)
5)

47.

49.

Seala cul de los siguientes sntomas NO se incluye entre los


sntomas de primer rango de la esquizofrenia:
1)
2)
3)
4)
5)

46.

1)
2)
3)
4)
5)

Antecedentes familiares.
Problemas obsttricos.
Consumo de txicos.
Infecciones virales.
Enfermedades infantiles.

Un varn de 24 aos es llevado a Urgencias por presentar gran


angustia. Sus padres comentan que llevan notndole raro desde
hace unos 8 meses; ha dejado de salir con sus amigos y de estudiar,
y parece encerrado en s mismo. Los dos ltimos meses tiene una
mirada rara, segn su madre, quien lo ha encontrado hablando
slo en numerosas ocasiones. En Urgencias el paciente te comenta
angustiado que est siendo sometido a una vigilancia y persecucin
asxiante por todas aquellas personas que van en los coches con
matrcula acabada en M, pues esa letra signica Morirs pronto.
Cmo se denomina este ltimo fenmeno psicopatolgico?
1)
2)
3)
4)
5)

45.

Presencia de crisis alucinatorias y delirantes.


Duracin superior a 6 meses.
Ausencia de factores psicosociales precipitantes.
Cambios afectivos y reduccin de la sociabilidad.

Los antipsicticos tpicos son todos igual de ecaces.


La potencia de un antipsictico se reere a su accin antidopaminrgica.
Cuanto ms potente es un antipsictico ms hipotensin
produce.
Los efectos extrapiramidales de los antipsicticos de baja
potencia son escasos.
La galactorrea se produce por afectacin del sistema tuberoinfundibular.

El mecanismo de accin que parece aumentar la ecacia de la


risperidona y reducir sus efectos extrapiramidales es:
1)
2)

Bloqueo de la recaptacin de dopamina.


Antagonismo del receptor H1.

Tfno. (0034) 91 782 43 30/33/34 E-mail: secretaria@ctomedicina.com www. ctomedicina.com

Test

1. vuelta Distancia

Psiquiatra
3)
4)
5)
54.

Tras una inyeccin de un psicofrmaco un paciente presenta un


movimiento involuntario de los ojos hacia arriba, que le resulta
doloroso y se mantiene a pesar de sus esfuerzos por evitarlo;
qu tratamiento precisa?
1)
2)
3)
4)
5)

55.

Potenciacin de la funcin GABArgica.


Bloqueo D2 y antagonismo 5HT2a.
Estimulacin D1 y D4.

1)
2)
3)
4)
5)
59.

Biperideno.
Amantadina.
Dantroleno.
Lorazepam.
Propranolol.

Un varn de 48 aos con larga historia de dependencia crnica


del alcohol, es ingresado en el Servicio de Psiquiatra porque
me persiguen homosexuales que me llaman perverso (sexual).
Al pasar por delante de los bares oye comentarios en su interior
que dicen que es marica, travestido y que van a ir a por l. Usted
pensar que el paciente presenta una alucinosis alcohlica si
detecta todo lo siguiente, EXCEPTO:
1)
2)

Un paciente esquizofrnico, de 25 aos, lleva 2 semanas tomando


15 mg/da de haloperidol. Los sntomas psicticos han mejorado
ostensiblemente. El paciente reere encontrarse muy inquieto,
no puede permanecer sentado mucho rato y se tiene que poner a
caminar sin rumbo. Cul, entre las siguientes, sera una medida
INCORRECTA?

3)
4)
5)

1)
2)
3)
4)
5)

56.

Aadir 1 mg de lorazepam dos veces al da.


Aadir 40 mg de propranolol una vez al da.
Sustituir el haloperidol por 2 mg de risperidona.
Disminuir la dosis de haloperidol a 10 mg/da.
Tranquilizar al paciente de lo transitorio de esta sintomatologa.

60.

1)
2)
3)
4)
5)

Terapia de modicacin de conducta.


Psicoeducacin.
Grupos de autoayuda de familias.
Psicoterapias dinmicas.
Centros de rehabilitacin psicosocial.

61.

57.

Dentro de las complicaciones asociadas al consumo agudo de


alcohol, hay una que est expresada de forma INCORRECTA:
1)
2)
3)
4)
5)

62.

La intoxicacin idiosincrsica se produce tras un consumo


mnimo de alcohol.
Existe buena correlacin entre la alcoholemia y los efectos
neuropsiquitricos.
El tratamiento de la intoxicacin es sintomtico.
La sobredosis de alcohol puede ser letal.
Con frecuencia vemos hiperglucemia como consecuencia
de la intoxicacin.

Un paciente ingresado por un traumatismo craneal presenta en


la noche del primer da de ingreso un episodio de agitacin psicomotriz, encontrndose sudoroso y con temblor grosero; dice
estar viendo cientos de insectos subiendo por las sbanas de la
cama y en la exploracin se encuentra desorientado en tiempo
y espacio; uno de los hallazgos siguientes sera INCOMPATIBLE
con el cuadro que padece:

1)
2)

CTO Medicina C/Francisco Silvela, 106 28002 - Madrid

4)
5)
63.

Naloxona tratamiento de la intoxicacin (sobredosis).


Metadona tratamiento de mantenimiento con opiceos.
Naltrexona tratamiento de deshabituacin con antagonistas.
Clonidina tratamiento de deshabituacin sin opiceos.
Metadona - tratamiento de la abstinencia (desintoxicacin).

Un paciente aparece en su consulta demandando tratamiento


pues presenta dolores generalizados, sudoracin, hipertermia,
nuseas y vmitos; a la exploracin detectamos marcas cutneas
en forma de ral y cicatrices de abscesos subcutneos en brazos
y piernas; no parecen existir antecedentes mdicos relevantes,
qu actitud deberemos tomar?

3)
58.

Naltrexona.
Diazepam.
Tiaprida.
Acamprosato.
Cianamida.

De las relaciones siguientes entre frmacos y su uso en la dependencia de opiceos, una es FALSA:
1)
2)
3)
4)
5)

TRASTORNOS
RELACIONADOS CON SUSTANCIAS

El nivel de conciencia del paciente no est alterado.


Las alucinaciones que presenta son predominantemente
auditivas.
El paciente ha bebido ms de lo habitual durante los
dos ltimos das.
El paciente desarrolla durante el ingreso un delirio ocupacional.
No hay ninguna alteracin somtica concomitante.

Dentro de las alternativas farmacolgicas para el tratamiento


de deshabituacin a largo plazo del paciente alcohlico, NO
utilizamos:
1)
2)
3)
4)
5)

Dentro de las opciones de abordaje de tipo psicosocial de la


esquizofrenia, se considera DESCARTADO en el momento actual:

VCM elevado.
GGT mayor de 300.
Descenso de la CDT (desialotransferrina).
Hipertermia.
Convulsiones tnico-clnicas.

Remitir al paciente a la Unidad de Hospitalizacin Psiquitrica


ms cercana.
Derivar al paciente a un centro especializado en drogodependientes.
Administrarle midazolam intravenoso como medicacin
sedante.
Ponerle un goteo de naloxona tras 2 ampollas i.v. iniciales.
Iniciar tratamiento con naltrexona y clonidina.

Cul de los siguientes frmacos NO se usa para intentar


reducir el riesgo de recadas en consumidores crnicos de
cocana?

Tfno. (0034) 91 782 43 30/33/34 E-mail: secretaria@ctomedicina.com www. ctomedicina.com

Test

1. vuelta Distancia

Psiquiatra
1)
2)
3)
4)
5)

Bromocriptina.
Desipramina.
Propranolol.
L-dopa.
Amantadina.

1)
2)
3)
4)

TRASTORNOS COGNOSCITIVOS: DELIRIUM, DEMENCIAS


Y AMNESIAS
64.

Un varn de 75 aos es ingresado para una reseccin prosttica transuretral; la primera noche tras la operacin se muestra
inquieto, creyendo reconocer en las enfermeras a los compaeros
que tuvo durante el Servicio Militar e intentando levantarse de
su cama; al ser retenido por la fuerza comienza a gritar llamando
a la Legin pues dice estar secuestrado; qu NO ser lgico
encontrar acompaando a este sndrome?
1)
2)
3)
4)
5)

65.

Tioridacina.
Sulpirida.
Zuclopentixol.
Haloperidol.
Clorpromacina.

70.

Hemograma.
TC craneal.
Test de Rorscharch.
Anlisis de orina.
ECG.

71.

A su consulta acude un hombre de 65 aos acompaado por sus


familiares, que reeren haber observado en el paciente desde
hace un mes una prdida sustancial en la memoria reciente y a
largo plazo. El propio enfermo se queja de la prdida de memoria
y la resalta con gran angustia, dando la impresin de que exagera
las deciencias. Usted se plantea el diagnstico diferencial entre
demencia y pseudodemencia. Acerca del mismo, es FALSO que:

Esquizoide.
Esquizoafectiva.
Esquizofrnica.
Esquizotpica.
Esquizofreniforme.

Una adolescente se desmaya en clase de gimnasia, tras lo que


es llevada al botiqun del instituto; all se detecta una tensin
arterial de 70/45 mmHg; la chica est muy delgada, detalle que
no habamos notado hasta la exploracin fsica; confrontada con
estos datos, cul de las siguientes conductas NO sera lgica
en esta paciente?
1)
2)
3)
4)
5)

72.

Personalidad obsesiva.
Personalidad antisocial.
Personalidad pasivo-agresiva.
Personalidad lmite.
Personalidad depresiva.

Una persona que lleva una vida bastante marginal, con un repertorio limitado de intereses, una llamativa ausencia de relaciones
sociales y con una respuesta emocional escasa, presenta una
personalidad:
1)
2)
3)
4)
5)

Hemograma y bioqumica plasmtica.


Niveles de hormonas tiroideas.
TC craneal.
Polisomnografa (estudio de la latencia REM).
Mini examen cognoscitivo (MEC) de Lobo.

CTO Medicina C/Francisco Silvela, 106 28002 - Madrid

Una chica de 17 aos es llevada al Servicio de Urgencias por sus


padres porque la han encontrado en su habitacin realizndose
cortes en los antebrazos. La paciente dice hacerse los cortes para
sentir que existo y expresa sentimientos crnicos de vaco. Es
conocida en el Servicio de Urgencias por los numerosos intentos
autolticos que ha realizado mediante la ingestin de pastillas.
Reere tener atracones ocasionalmente y sus padres estn
asustados por su promiscuidad sexual. Durante la entrevista,
dice en ocasiones que se quiere ir de vacaciones, para poco
despus ponerse a llorar y expresar deseos de suicidio. Cul
es el trastorno de personalidad que padece?
1)
2)
3)
4)
5)

Una paciente de 70 aos acude a su consulta reriendo frecuentes


prdidas de objetos cotidianos y ocasional desorientacin en
calles poco familiares; qu prueba de entre las siguientes NO
utilizara en el estudio inicial de este caso de deterioro cognitivo?
1)
2)
3)
4)
5)

68.

69.

Empeoramiento nocturno.
Alucinaciones visuales.
Agitacin psicomotriz.
Preservacin de la capacidad de atencin.
Fluctuaciones de la clnica.

En un hospital numerosos enfermos se encuentran en riesgo de


sufrir un delirium; en el estudio de este sndrome NO resulta til
una de las siguientes pruebas:
1)
2)
3)
4)
5)

67.

OTROS TRASTORNOS MENTALES: PERSONALIDAD,


ALIMENTACIN, SUEO, INFANTILES

Qu antipsictico de los siguientes se recomienda para el


control de la agitacin psicomotriz en los casos de delirium?
1)
2)
3)
4)
5)

66.

5)

Las alteraciones de las pruebas de neuroimagen sean altamente inespeccas en ambos cuadros.
Los pacientes depresivos tienden a mejorar por la noche,
mientras que los dementes suelen hacerlo por la maana.
Los pacientes con demencia suelen exagerar sus fallos
cognitivos y los depresivos a minimizarlos.
Los cambios en las pruebas neuropsicolgicas son incongruentes en la depresin y consistentes en las demencias.
La mitad de las llamadas pseuodemencias depresivas
desarrollarn, a medio o largo plazo, una verdadera
demencia.

Preocupacin por el aspecto fsico y la esttica.


Alto rendimiento acadmico.
Tendencia a vestirse con ropas ajustadas y ligeras.
Uso excesivo de laxantes.
Evitacin de las reuniones sociales en donde pueda verse
forzada a comer en pblico.

Dentro de las caractersticas del atracn bulmico, NO se


encuentra:

Tfno. (0034) 91 782 43 30/33/34 E-mail: secretaria@ctomedicina.com www. ctomedicina.com

Test

1. vuelta Distancia

Psiquiatra
1)
2)
3)
4)
5)
73.

Al revisar la polisomnografa de su paciente encuentra un


perodo con ondas muy lentas en el EEG, asocindose con un
tono muscular bajo y sin movimientos oculares; qu trastorno
del sueo se asocia a esta fase?
1)
2)
3)
4)
5)

74.

Estereotipias motoras.
Relativa preservacin del lenguaje.
Escaso contacto ocular.
Desinters por las relaciones con otros nios.
Intolerancia a los cambios de la rutina diaria.

En uno de los siguientes trastornos de la infancia, el tratamiento


con antipsicticos es de eleccin:
1)
2)
3)
4)
5)

77.

Betabloqueantes.
Clonazepam.
Carbamacepina y valproico.
Clomipramina.
Sistemas de ventilacin positiva.

Dentro del autismo infantil, es frecuente observar alteraciones


conductuales, pero NO es tpico:
1)
2)
3)
4)
5)

76.

Bruxismo.
Apnea obstructiva del sueo.
Enuresis.
Sonambulismo.
Pesadillas.

La prdida brusca del tono muscular en respuesta a una emocin


es un sntoma casi patognomnico de un trastorno; cul es el
tratamiento del mismo?
1)
2)
3)
4)
5)

75.

Tensin creciente antes del atracn.


Relacin desproporcionada ingesta/tiempo.
Sensacin de prdida de control.
Seleccin de alimentos hipocalricos.
Aumento de la frecuencia por la noche.

Dcit de atencin con hiperactividad.


Autismo.
Mutismo selectivo.
Sndrome de la Tourette.
Enuresis nocturna.

Carlos es un nio de 7 aos que sufre con frecuencia castigos


en clase por no aceptar la disciplina; pasa las horas de colegio
levantndose continuamente del pupitre y molestando a sus
compaeros, que han comenzado a hacerle el vaco pues dicen
que es muy bruto; en casa se comporta de forma parecida,
habiendo destrozado todos sus juguetes y sufriendo con frecuencia accidentes, pues no mide correctamente el riesgo; qu
problema presenta con mayor probabilidad?
1)
2)
3)
4)
5)

Retraso mental.
Trastorno disocial de la personalidad.
Conducta oposicionista.
Dcit de atencin con hiperactividad.
Depresin enmascarada.

CTO Medicina C/Francisco Silvela, 106 28002 - Madrid

Tfno. (0034) 91 782 43 30/33/34 E-mail: secretaria@ctomedicina.com www. ctomedicina.com

Comentarios de Test a distancia 1. vuelta

Psiquiatra

Comentarios de Test a distancia

1. vuelta

Psiquiatra
pulsivos), pero no vemos lo contrario (un paciente con crisis de angustia
no desarrolla fenmenos obsesivo-compulsivos).

TRASTORNOS NEURTICOS
Pregunta 1.-R: 2
Los trastornos primarios por ansiedad se pueden presentar de dos formas:
Continua, como en el trastorno por ansiedad generalizada.
Episdica, en forma de crisis; cuando las crisis tienen lugar siempre
al enfrentarse a una determinada situacin decimos que el paciente
sufre una fobia; otros pacientes tienen crisis espontneas (llamadas
crisis de angustia o ataques de pnico), sin relacin aparente con
estmulo alguno; cuando estas crisis espontneas se repiten y repercuten en la vida de las personas, dando lugar a la aparicin de
complicaciones psicolgicas, hablamos de trastorno por angustia
(o trastorno de pnico).
No es raro que los trastornos por ansiedad se vayan confundiendo entre
s cuando se cronican o complican; el ejemplo ms claro lo vemos en
las complicaciones que surgen cuando el trastorno por angustia se
prolonga en el tiempo.

Pregunta 2.-R: 1
En el trastorno por angustia (pnico) diferenciamos tres necesidades
de tratamiento:
El control de los sntomas agudos de una crisis de angustia (ataque
de pnico) se realiza con benzodiacepinas por va oral o sublingual
(aunque en Espaa NO se comercializa ninguna presentacin
sublingual de benzodiacepinas). Es igualmente necesario ayudar
al paciente a frenar la hiperventilacin y explicarle la ausencia de
consecuencias fsicas de la crisis y las caractersticas de su trastorno.
La prevencin de nuevas crisis se realiza con antidepresivos, siendo
hoy en da de eleccin los ISRS (paroxetina o citalopram son los ms
utilizados) por su mejor perl de efectos secundarios; sto no quiere
decir que no sean de utilidad los antidepresivos clsicos (tricclicos
o IMAO). Tambin son ecaces las benzodiacepinas de alta potencia
(alprazolam, clonazepam, lorazepam), pero el miedo a la posible dependencia ha hecho que se trate de limitar su uso a las primeras semanas
del tratamiento, mientras el antidepresivo alcanza su mxima ecacia.
Finalmente, resulta imprescindible evaluar las distintas complicaciones
psicolgicas o psiquitricas del trastorno por angustia/pnico y ofrecer
un tratamiento especco si persisten una vez controladas las crisis.
La buspirona es un ansioltico NO benzodiacepnico (agonista parcial
serotoninrgico) con un perl ms adecuado para el trastorno por
ansiedad generalizada aunque su potencia ansioltica sea baja; carece
de efectos sedantes (no sirve para cortar una crisis de ansiedad), no
produce dependencia ni abstinencia y no interacciona con el alcohol;
tiene un inicio de accin lento (semanas) y no ha demostrado efectos
preventivos antipnico.

Pregunta 1. Complicaciones del trastorno de pnico.


De una forma algo articial, la clasicacin DSM tambin incluye dentro
de los trastornos por ansiedad los trastornos obsesivo-compulsivos (en
donde la ansiedad es secundaria a los fenmenos obsesivos y compulsivos) y los trastornos por estrs agudo y postraumtico (en donde la
ansiedad es uno de los muchos sntomas que aparecen ante una situacin
estresante de intensidad mxima).
En el trastorno obsesivo-compulsivo, los pacientes pueden sufrir crisis
de ansiedad (sobre todo cuando se les impide realizar los rituales comCTO Medicina C/Francisco Silvela, 106 28002 - Madrid

VENTAJAS

DESVENTAJAS

Antidepresivos

No producen
dependencia
Tratan la posible
depresin asociada

Tardan en hacer efecto (semanas)


Pueden producir un aumento
inicial de la ansiedad (empezar a
dosis bajas y subir lentamente)

Benzodiacepinas

Actan de forma
rpida (das)
Escasos efectos
adversos (sedacin
transitoria)

Riesgo de abuso y dependencia

Pregunta 2. Ventajas y desventajas de los tratamientos


preventivos para el pnico.

Tfno. (0034) 91 782 43 30/33/34 E-mail: secretaria@ctomedicina.com www. ctomedicina.com

Comentarios de Test a distancia 1. vuelta

Psiquiatra
Pregunta 3.-R: 3
La mayora de los pacientes con agorafobia presentan adems crisis de
angustia espontneas (ataques de pnico); se considera, por tanto, que
su agorafobia es secundaria. Pero tambin hay formas de agorafobia
primaria en las que no detectamos crisis espontneas. El tratamiento de
la agorafobia en s no es distinto: tcnicas psicolgicas de modicacin
de conducta (sobre todo la exposicin en vivo). La diferencia vendr dada
por la necesidad de aadir tratamiento farmacolgico para prevenir las
crisis de angustia en el primer caso. Las tcnicas psicolgicas basadas en
la modicacin de conducta son el tratamiento de eleccin en todas las
fobias. Suele realizarse primero un listado de todas las situaciones temidas
y una ordenacin en funcin del miedo asociado (jerarquizacin); posteriormente se programan ejercicios de exposicin en vivo, comenzando
en las situaciones ms fciles. Se han desarrollado otras tcnicas ms
agresivas que buscan la exposicin brusca a la situacin fbica en su
mxima intensidad (inundacin o implosin), pero son peor toleradas.

de impulsin (en las que el paciente tiene miedo de llevar a cabo esas
ideas tan desagradables y evita situaciones potencialmente peligrosas
como el uso de cuchillos, el acercarse a las ventanas o el aproximarse
a los andenes del tren por el miedo a presentar un impulso frente al
que no se pueda resistir).

Pregunta 4.-R: 3
El paciente con fobia social tiene miedo de aquellas situaciones en las
que se expone a la opinin de otras personas. Lo que le preocupa no
es, por tanto, el lugar en el que se encuentra, sino la reaccin de los
dems ante su comportamiento. No es raro que estos pacientes limiten
sus actividades sociales, pudiendo confundirse con otros pacientes que
tambin presentan retraccin social (agorafbicos que no salen de sus
casas por el miedo a los desplazamientos, esquizofrnicos o depresivos
que restringen sus relaciones sociales por la prdida de inters asociada
a sus enfermedades, personalidades esquizoides sin inters real por la
conducta social). Quizs lo ms complicado sea diferenciarles de las personalidades evitativas o fbicas, con quienes comparten los mismos errores
de pensamiento (preocuparse en exceso por la opinin de los dems),
si bien se supone que la fobia social comienza en un momento dado
(en torno a los 20 aos) y las personas evitativas siempre han sido as.
En las fobias sociales los frmacos pueden ayudar a los tratamientos
psicolgicos de dos formas distintas; en las formas de fobia social restringidas a situaciones poco frecuentes (p. ej., miedo a hablar en pblico
delante de un auditorio) los bloqueantes de los receptores betaadrenrgicos (propranolol, atenolol) disminuyen la respuesta ansiosa perifrica
y pueden permitir un mayor autocontrol de la ansiedad, aunque el
tratamiento de eleccin sera psicolgico, buscando mejorar la capacidad de exponer en pblico; en la forma generalizada de fobia social
se propone el tratamiento con antidepresivos como coadyuvantes de
la psicoterapia. En los ltimos aos los ISRS han desplazado a los IMAO
en estos pacientes con fobia social grave.

Aunque en muchos casos los fenmenos compulsivos guardan una


relacin ms o menos lgica con las ideas obsesivas, en otros pacientes
veremos rituales totalmente independientes, con una relacin ms bien
mgica (p. ej., la necesidad de colocar las cosas en un determinado
orden para que no suceda nada malo).

Trastornos por tics (Tourette)


Trastornos del control de los impulsos (juego patolgico/ludopata,
cleptomana, piromana, tricotilomana, trastorno explosivo intermitente)
Trastornos de la conducta alimentaria
Algunos trastornos somatomorfos (hipocondra, dismorfofobia)
Algunos trastornos de la personalidad (esquizotpico, lmite)
Algunas formas de toxicomana
Algunas parafilias
Algunos trastornos generalizados del desarrollo (Asperger, autismo atpico)

Pregunta 5. Presentacin clnica de los trastornos obsesivo-compulsivos.

Pregunta 5.-R: 4
Las dos principales formas de presentacin del trastorno obsesivocompulsivo son:
La combinacin de ideas obsesivas relacionadas con la posible
contaminacin o contagio de enfermedades y rituales compulsivos
de lavado excesivo o de evitacin del contacto con posibles fuentes
contaminantes.
La combinacin de ideas obsesivas de duda o incertidumbre y rituales
compulsivos de comprobacin.

Pregunta 6.-R: 5
El trastorno obsesivo-compulsivo idioptico suele aparecer al nal de
la adolescencia o principio de la juventud, de una forma insidiosa, y
evoluciona con carcter crnico y de forma uctuante, agravndose en
situaciones de estrs. Es raro que desaparezca por completo, pero tambin es excepcional que alcance una gravedad invalidante. La mayora
de los pacientes acaban por acostumbrarse a los fenmenos obsesivos
y compulsivos, lo que puede explicar su escasa presencia en consulta
a pesar de su elevada prevalencia en poblacin general (cerca del 2%).
Cuando debuta en la infancia se puede asociar con los trastornos por
tics (tanto los tics motores crnicos como el trastorno de la Tourette).
Recientemente se ha descrito una asociacin entre algunas formas
infantiles de inicio agudo y fenmenos autoinmunes cercanos a la
ebre reumtica (secundarios, por tanto, a infecciones estreptoccicas
[estreptococo betahemoltico o SGA] farngeas); se ha denominado
PANDAS (siglas en ingls del trastorno neuropsiquitrico peditrico de
origen autoinmune asociado al estreptococo) y se supone que comparte
un mismo mecanismo patognico con la corea de Sydenham (lesin
mediada por autoanticuerpos frente a los ganglios basales).
Cuando el trastorno obsesivo-compulsivo aparece en adultos o ancianos
es necesario buscar problemas neurolgicos como causa del mismo
(corea de Huntington, accidentes vasculares cerebrales, etc.).
En cualquier caso, tanto en las formas idiopticas como en las formas
secundariasse implica a las mismas regiones cerebrales, loscircuitosde
conexin entre corteza prefrontal, ganglios basales y tlamo. De hecho,
la posible utilidad de la psicociruga en algunos casos seleccionados se
basa en la interrupcin de alguna de las vas de estos circuitos.
El neurotransmisor ms relacionado con el trastorno obsesivo-compulsivo es la serotonina, lo que va a ser utilizado en el tratamiento
farmacolgico.

Menos frecuentes son las obsesiones centradas en la necesidad de


orden y simetra, que dan lugar a compulsiones de colocacin y
ordenacin. Sin embargo, no es raro encontrar pacientes con otros
fenmenos ms extraos, destacando las llamadas ideas de contraste
(ideas de contenidos agresivos o sexuales totalmente inapropiadas para
la situacin en la que se encuentra el paciente y en desacuerdo con
sus deseos o creencias) que pueden conducir a la aparicin de fobias

Pregunta 7.-R: 5
Uno de los datos ms sorprendentes del trastorno obsesivo-compulsivo
es su respuesta selectiva a medicacin serotoninrgica; el tratamiento
farmacolgico de eleccin son los antidepresivos serotoninrgicos,
pudiendo en ocasiones potenciarse su efecto con otras sustancias (litio,
buspirona). En los casos refractarios y en aquellos en los que coexistan
tics puede ser til aadir un antipsictico.

CTO Medicina C/Francisco Silvela, 106 28002 - Madrid

Tfno. (0034) 91 782 43 30/33/34 E-mail: secretaria@ctomedicina.com www. ctomedicina.com

Comentarios de Test a distancia 1. vuelta

Psiquiatra
Dentro de los antidepresivos, el mejor estudiado es el antidepresivo
tricclico clorimipramina (o clomipramina), si bien sus efectos adversos
le han relegado a un segundo plano tras la aparicin de los ISRS. stos se
han convertido en la medicacin de primera eleccin, teniendo en cuenta
adems que en este trastorno suelen necesitarse dosis dos o tres veces
superiores a las habituales para la depresin y tiempos de tratamiento
muy prolongados, lo que disminuye la efectividad del tratamiento con
clorimipramina al aumentar los abandonos. Los IMAO se han usado
para pacientes resistentes, mientras que las benzodiacepinas carecen
de efecto antiobsesivo especco.
Junto con el tratamiento farmacolgico es necesario un tratamiento
psicolgico basado (como en las fobias) en las tcnicas de modicacin
de conducta; en concreto, la tcnica preferida se denomina exposicin (a
la situacin obsesiva temida) con prevencin de respuesta (compulsiva).
El psicoanlisis no obtiene resultados en los pacientes con este trastorno.
En casos refractarios al tratamiento farmacolgico, la psicociruga (cingulotoma, capsulotoma bilateral anterior, tractotoma subcaudada)
puede ser ecaz.
Pregunta 8.-R: 3
Podemos agrupar los sntomas del trastorno por estrs postraumtico
en cuatro tipos:
Reexperimentacin del acontecimiento: pesadillas, imgenes tipo
ash-back, recuerdos intrusivos.
Evitacin voluntaria (fbica) o involuntaria (amnesia psicgena) de
todo lo relacionado con el acontecimiento.
Estado de hiperalerta: reacciones bruscas ante los ruidos, problemas
de concentracin, insomnio, irritabilidad.
Cambios emocionales: embotamiento, sensacin de corte vital.
Cuando el trastorno por estrs no ha alcanzado un mes de duracin se
denomina trastorno por estrs agudo, y suelen predominar los sntomas emocionales y disociativos. Se diagnostica un trastorno de estrs
postraumtico cuando los sntomas superan el mes de duracin, considerndose que se cronica cuando alcanza los 3 meses de duracin.
Se describen formas de inicio demorado que comienzan pasados varios
meses del suceso, generalmente tras la exposicin a alguna situacin
que reactiva su recuerdo.
Dentro de las complicaciones de estos trastornos destacan los sndromes
depresivos y el abuso de sustancias.
De las posibles situaciones traumticas, aquellas que se deben a fenmenos naturales (catstrofes como terremotos o inundaciones) producen menos problemas que las que se deben a la intervencin humana;
dentro de stas, los accidentes son menos traumticos que los delitos,
en los que una persona provoca voluntariamente un dao sobre otra; los
delitos sexuales y los secuestros producen estrs postraumtico en un
nmero muy elevado de casos (ms del 80% de las vctimas de violacin).
No existe un tratamiento especco del estrs postraumtico; los antidepresivos (IMAO, ISRS) parecen obtener mejores resultados cuando
predominan los fenmenos de reexperimentacin y los cambios
emocionales; las conductas de evitacin precisarn de un tratamiento
conductual adecuado; para reducir el nivel de alerta se pueden usar
benzodiacepinas (cuidado con el abuso de sustancias) y tcnicas de
relajacin. Lo que s parece clara es la necesidad de detectar y tratar
precozmente a estos pacientes, pues la cronicacin de los sntomas
predice una mala respuesta al tratamiento.
Pregunta 9.-R: 3
Dentro de los pacientes que se presentan con sntomas fsicos para
los que no se encuentra una explicacin mdica (SOMATOMORFOS)
tenemos:
CTO Medicina C/Francisco Silvela, 106 28002 - Madrid

Somatizadores: se quejan de numerosos sntomas (digestivos,


neurolgicos, genitales, cardiorrespiratorios); tienden a cambiar
de mdico cuando se les confronta con la posibilidad de un origen
psicolgico de sus molestias; pueden abusar de la automedicacin
(analgsicos) y corren el riesgo de someterse a pruebas diagnsticas
innecesarias y peligrosas; en su mayora son mujeres, que suelen
comenzar a tener sntomas en la juventud, siguiendo un curso
crnico y uctuante; se utiliza a veces en estos casos el epnimo
de sndrome de Briquet. Una variante de somatizador ms limitada
en su expresin clnica es el paciente que se queja slo de dolor,
siendo ste desproporcionado para los hallazgos exploratorios.
Hipocondracos y dismorfofbicos: estn preocupados por padecer
una enfermedad o sufrir un defecto fsico, respectivamente; aunque
tambin pueden cambiar de mdico si no obtienen la atencin que
creen necesaria, es ms raro en ellos la tendencia a la automedicacin o la realizacin de pruebas complementarias de riesgo; en el
caso de los dismorfofbicos, el peligro fundamental son las posibles
intervenciones estticas (mdicas o quirrgicas) que pueden dejar
secuelas; en ambos casos no veremos un predominio de ningn sexo
y el inicio suele ser ms tardo; los pacientes oscilan en la intensidad
de sus preocupaciones, desde formas cercanas a los trastornos
obsesivo-compulsivos hasta formas casi delirantes. Se propone para
estos pacientes un tratamiento similar al de los trastornos obsesivocompulsivos (ISRS, clorimipramina), asociado a veces a antipsicticos.
Conversivos: se suelen presentar en Urgencias con signos neurolgicos incongruentes a la exploracin fsica (convulsiones, parlisis,
anestesias, cegueras); casi siempre se identica un factor estresante
relacionado temporalmente con el inicio de los sntomas; tienden a la
recuperacin espontnea, pero tambin a la recurrencia; en algunos
raros casos veremos formas de evolucin crnica; como en el caso
de los somatizadores, suelen debutar en la juventud y hay un claro
predominio de las mujeres; en los pases desarrollados son cada vez
menos frecuentes. Parece que con el desarrollo cultural se produce
una mayor tendencia a la somatizacin como expresin fsica del
malestar psicolgico; ambas, conversin y somatizacin, junto con las
variantes disociativas, componen las distintas formas de presentacin
de la histeria, una enfermedad clsica dentro de la Psiquiatra que
parece traducir a sntomas fsicos o psiquitricos incongruentes la
respuesta de determinadas personas ante el malestar psicolgico.

Conviene NO confundir a estos pacientes (en los que los sntomas son
involuntarios y no existe enfermedad fsica real) con los pacientes PSICOSOMTICOS (que presentan una enfermedad fsica real e involuntaria,
en donde los factores psicolgicos inuyen en su origen o evolucin) ni
con los pacientes que ngen VOLUNTARIAMENTE tener una enfermedad.
Dentro de estos ltimos diferenciamos los simuladores, en los que existe
una motivacin evidente de tipo econmico o legal, de los trastornos
facticios, en los que la motivacin parece ser de tipo psicolgico, la necesidad de asumir la identidad de enfermo, incluso a costa de perjuicios
para su salud o su economa. La forma ms espectacular de trastorno
facticio es el sndrome de Mnchausen, en el que los sntomas inventados
y los signos autoprovocados abarcan diferentes rganos y sistemas; la
mayora de los facticios, sin embargo, son formas limitadas a sntomas
ms concretos, siendo ms difciles de detectar, pues los pacientes se
preocupan de no ser descubiertos y de parecerse lo ms posible a la
enfermedad real. Desde el punto de vista legal, la simulacin es un delito
evidente; los trastornos facticios pueden convertirse en un problema
legal cuando los pacientes provocan la enfermedad en terceras personas
(hijos, esposo) para poder asumir la identidad de cuidador (trastornos
facticios por poderes). En la mayora de estos pacientes encontramos
graves alteraciones de la personalidad, siendo refractarios al tratamiento.

Tfno. (0034) 91 782 43 30/33/34 E-mail: secretaria@ctomedicina.com www. ctomedicina.

Comentarios de Test a distancia 1. vuelta

Psiquiatra
BZD tendrn mayor o menor potencia, pero a las dosis adecuadas
todas comparten los efectos anticonvulsivos, ansiolticos, hipnticos
y miorrelajantes. Sus principales diferencias son farmacocinticas
(velocidad de absorcin, vida media de eliminacin, presencia o no
de metabolizacin heptica).

Pregunta 9. Caractersticas de los fenmenos histricos.


Pregunta 10.-R: 5
En los trastornos disociativos los pacientes presentan sntomas psicolgicos (NO fsicos) extraos como cambios de comportamiento, amnesias
o viajes sin sentido que no encajan en las enfermedades psiquitricas o
neurolgicas habituales y que responden de forma impredecible ante
los tratamientos habituales.
Como en el caso de los pacientes conversivos, es frecuente encontrar
un factor estresante relacionado temporalmente con el sntoma y una
tendencia a la desaparicin espontnea y a la recurrencia. Los sntomas
disociativos pueden aparecer en el contexto de otras enfermedades
psiquitricas, destacando su asociacin con los trastornos por estrs
(agudo y postraumtico) y con el trastorno de personalidad lmite. En
los ltimos aos se insiste en la frecuencia de antecedentes de abuso
sexual en la infancia de los pacientes con trastornos disociativos graves.
Dependiendo de la cultura del pas encontramos diferentes formas de
trastornos disociativos; as, las ms frecuentes en Espaa parecen ser los
episodios de amnesia, los ataques de nervios y el estupor psicgeno,
siendo excepcionales las fugas y anecdticos los casos de alteraciones
disociativas de la personalidad (personalidad mltiple); en pases menos desarrollados veremos numerosos casos de trances y posesiones
disociativas, y en EE.UU. llama la atencin la elevada frecuencia de casos
referidos con alteraciones disociativas de la personalidad.
La clasicacin DSM (no la CIE-10) incluye dentro de los trastornos disociativos a los cuadros de despersonalizacin y desrealizacin crnicas;
no hay que olvidar que estos sntomas de extraeza ante uno mismo o
el entorno pueden aparecer en numerosas enfermedades psiquitricas
(trastornos por ansiedad, sobre todo en crisis, sndromes depresivos,
psicosis agudas, intoxicaciones por drogas) y en algunos trastornos neuropsiquitricos, sobre todo los que afectan al lbulo temporal (epilepsia
parcial compleja, tumores, encefalitis); los casos en donde aparecen de
forma aislada y con evolucin crnica son excepcionales.
Pregunta 11.-R: 3
Las benzodiacepinas (BZD) interaccionan con unos lugares de accin
especcos (a veces llamados receptores benzodiacepnicos u omega),
dentro del complejo GABA-A; estos lugares de accin estn formados
por diversas subunidades proteicas, para las que existen numerosas
variantes, sin que se conozca todava la importancia funcional de cada
una de ellas. Desde el punto de vista farmacodinmico, todas las BZD
hacen lo mismo: se unen a su lugar de accin y modican la anidad
del receptor gabargico de tipo A por el GABA, aumentando su actividad (modicacin alostrica positiva o potenciacin). Las diferentes
CTO Medicina C/Francisco Silvela, 106 28002 - Madrid

Pregunta 12.-R: 2
Una de las ventajas de que las BZD tengan un lugar especco de accin ha
sido la posibilidad de disear un antagonista especco capaz de revertir
sus efectos en el caso de sobredosis; el umacenilo es el tratamiento
de eleccin en las intoxicaciones por BZD y se ha convertido, junto con
la naloxona (antagonista de los receptores opioides y tratamiento de
eleccin en las intoxicaciones por opiceos), en una medicacin bsica
de los servicios de urgencias. En el caso del resto de psicofrmacos
carecemos de antdotos especcos.
Las sobredosis de antidepresivos varan en su gravedad en funcin del
tipo de antidepresivo, siendo los ISR los frmacos ms seguros y los
ADT los ms peligrosos (cardiotoxicidad), ocupando los IMAO un lugar
intermedio.
Las intoxicaciones con antipsicticos tienen un riesgo considerablemente
menor, dependiendo sus efectos en sobredosis de otras caractersticas
(bloqueo histaminrgico, bloqueo alfaadrenrgico, efectos anticolinrgicos) que varan enormemente de unos a otros; en el caso de presentarse
graves efectos extrapiramidales agudos, el tratamiento de eleccin son
los anticolinrgicos (biperideno, trihexifenidilo).
Dentro de los estabilizadores del humor, el litio es el ms peligroso en el
caso de sobredosicacin, aunque la principal causa de intoxicacin por
litio no va a ser la toma voluntaria o accidental de un exceso de medicacin,
sino las interacciones con frmacos que alteran su eliminacin renal o la
concurrencia de problemas hidroelectrolticos que tienen el mismo efecto;
el tratamiento de eleccin en caso de intoxicacin grave es la hemodilisis.
Pregunta 13.-R: 5
La mayora de las BZD se eliminan por va heptica mediante un doble
mecanismo: metabolizacin y conjugacin; el primero de estos procesos se afecta gravemente en el caso de hepatopata, permaneciendo el
segundo casi inalterado. Por eso las BZD, que slo se conjugan sin metabolizarse en el hgado, son las de eleccin en el caso de hepatopata (en
Espaa slo nos queda el lorazepam con una vida media de eliminacin
corta, cercana a las 8 horas).
VIDA MEDIA
Larga

Corta

nica

Mltiple

Rebote de la ansiedad entre las dosis


diurnas (con BZD de alta potencia)

No

Rebote matinal de la ansiedad o el


insomnio (con BZD de alta potencia)

No

Abstinencia

No

Restos diurnos en su uso como


hipntico (resaca)

No

Riesgo de acumulacin (en ancianos)

No

Dosis/da

Pregunta 13. Ventajas e inconvenientes de las benzodiacepinas


en funcin de su vida media.
Pregunta 14.-R: 1
Existen otras BZD, de alta potencia, que tampoco se metabolizan de
forma sustancial en el hgado (alprazolam, triazolam, midazolam) y cuya
vida media de eliminacin es quizs demasiado corta; en el caso de tria-

Tfno. (0034) 91 782 43 30/33/34 E-mail: secretaria@ctomedicina.com www. ctomedicina.com

Comentarios de Test a distancia 1. vuelta

Psiquiatra
zolam y midazolam es de menos de 6 horas por lo que no se usan como
ansiolticos; triazolam se utiliza para tratar el insomnio de conciliacin
(siempre en tandas cortas de pocas semanas para no producir tolerancia
y dependencia), y midazolam se usa como preanestsico y en procedimientos instrumentales potencialmente desagradables (cateterismos,
colonoscopias) para relajar a los pacientes.
Como ansioliticos

Betabloqueantes
Buspirona
Antidepresivos
Anticonvulsivos
Antipsicticos

Como hipnticos

Zolpidem, zoplicona, zalepln


Antihistamnicos
AD (y AP) sedantes

Como miorrelajantes

Ciclobenzaprina

Como anticonvulsivos

Fenobarbital, fenitona,
cido valproico, carbamacepina,
gabapentina, lamotriguina, topiramato

Pregunta 14. Alternativas a las benzodiacepinas


en sus diferentes indicaciones.

TRASTORNOS AFECTIVOS. FRMACOS ANTIDEPRESIVOS


Y ESTABILIZADORES DEL HUMOR
Pregunta 15.-R: 4
Clasicamos los sndromes afectivos en funcin de dos criterios: su
gravedad y su duracin. La gravedad se evala con diferentes escalas
de sntomas (depresivos o manacos), aunque el principal criterio de
gravedad ser la repercusin que el sndrome clnico tenga sobre el
funcionamiento del paciente. Atendiendo a la gravedad, diferenciamos
los sndromes depresivos o manacos en MAYORES y MENORES.
Para que un sndrome depresivo mayor sea considerado relevante se
exige una duracin mnima de 2 semanas (EPISODIO DEPRESIVO MAYOR), mientras que en el caso de la mana, la duracin se reduce a una
semana o menos, si precisa hospitalizacin por su intensidad (EPISODIO
MANACO). En los sndromes depresivos menores se exige una mayor
duracin, debiendo persistir los sntomas depresivos 2 aos para aceptar
el diagnstico de DISTIMIA; en los sndromes manacos s se acepta la
importancia de los cuadros menores de duracin recortada, diagnosticndose de HIPOMANA a los sndromes manacos menores que duran
al menos 4 das; la alternancia de sntomas hipomanacos y sntomas
depresivos menores durante un perodo de al menos 2 aos recibe el
diagnstico de CICLOTIMIA.
Los cuadros depresivos son mucho ms frecuentes que los manacos,
encontrndose en los primeros un claro predominio femenino, mientras que en los segundos hay una cierta igualdad. De igual forma, los
sndromes episdicos (de semanas o meses de duracin) son ms
frecuentes que los sndromes crnicos (de aos de duracin). De ah
que el trastorno afectivo ms frecuente sea el episodio depresivo mayor.
La melancola (o depresin endgena) es una variante de episodio
depresivo mayor que se identica por la presencia de unos sntomas
caractersticos; por tanto, aunque todos los casos de melancola son
episodios depresivos mayores, slo una pequea parte de los episodios
depresivos mayores presentan sntomas melanclicos.
Pregunta 16.-R: 4
Los episodios depresivos mayores (EDM) reciben diferentes adjetivos
calicativos en funcin de distintas caractersticas:
CTO Medicina C/Francisco Silvela, 106 28002 - Madrid

Segn la gravedad de los sntomas, podemos ver EDM de intensidad leve,


moderada o grave; las formas ms extremas de EDM llegan a presentar
sntomas psicticos (delirios, sobre todo), por lo que se habla de EDM
con sntomas psicticos o depresin psictica (o depresin delirante).
Segn el patrn de sntomas tenemos EDM con sntomas melanclicos (o depresin endgena) y EDM con sntomas atpicos; sin
embargo, la mayora de los EDM no encajan en ninguno de estos dos
patrones, presentando sntomas inespeccos; parece que con las
recadas sucesivas de la enfermedad los sntomas van adquiriendo
un carcter ms endgeno y pueden alcanzar con mayor facilidad
una gravedad psictica; lo mismo parece ocurrir en los EDM que
debutan en la edad avanzada. Tambin podemos ver un EDM con
sntomas catatnicos (o depresin estuporosa).
Cuando el inicio del EDM guarda una relacin con el puerperio se
habla de EDM de inicio puerperal o depresin puerperal (o depresin
posparto).

La depresin neurtica (trmino clsico) se corresponde ms o menos


con la distimia de la clasicacin actual, no con los EDM.
Pregunta 17.-R: 5
Los sntomas depresivos atpicos incluyen tres caractersticas que, en
teora, predeciran una mala respuesta a medicacin antidepresiva:
La presencia de rasgos patolgicos de personalidad, en la lnea
dependiente o histrinica.
La existencia de un factor estresante asociado al inicio de los sntomas
(frecuentemente un problema sentimental).
La ausencia de sntomas melanclicos, observndose una inversin de
los sntomas biolgicos (aparecen somnolencia excesiva y aumento
del apetito).
Sin embargo, se describe una buena respuesta al tratamiento con IMAO,
siendo inferiores los resultados obtenidos con otros antidepresivos. Esta
selectividad farmacolgica se ha relacionado con una mayor frecuencia
de alteraciones serotoninrgicas en estos pacientes. En general, los sntomas atpicos no implican una gravedad excesiva; de hecho, podemos ver
sntomas atpicos tanto en los EDM como en las distimias. En cambio, los
sntomas melanclicos s se asocian a una mayor gravedad (y a un mayor
riesgo de suicidio, por tanto); en estos pacientes melanclicos se encuentran
ms alteraciones neurobiolgicas, por lo que su respuesta a los tratamientos biolgicos es superior (tanto a antidepresivos como a electrochoque).
Pregunta 18.-R: 5
La depresin debuta en general en la edad adulta (sobre los 20 aos
debutan los bipolares, en torno a los 40 aos los depresivos), siendo raros
los casos de inicio en la infancia o la vejez. Adems, en esas edades el
diagnstico se complica al enmascararse la depresin bajo otros sntomas.
En los ancianos no es raro que se oculte una depresin tras una serie
de quejas fsicas o de memoria, lo que en parte puede reejar la mayor
preocupacin por estos sntomas en este grupo de edad. Tambin es
destacable la mayor frecuencia de sntomas psicomotores (inhibicin,
agitacin) y de sntomas psicticos; se han relacionado ambos sntomas
con alteraciones en los ganglios basales, que podran deberse al deterioro
cerebral asociado a la edad o a la presencia de factores de riesgo cerebrovascular. La presencia de sntomas psicticos implica la necesidad de
modicar el tratamiento, aadiendo antipsicticos a los antidepresivos
o recurriendo al electrochoque. No hay que olvidar que la edad es por
s misma un factor que aumenta el riesgo de suicidio, lo que, aadido
a la presencia de un sndrome depresivo y a la gravedad psictica, que
con frecuencia alcanza, explica el enorme riesgo de suicidio que tienen
las depresiones graves del anciano.

Tfno. (0034) 91 782 43 30/33/34 E-mail: secretaria@ctomedicina.com www. ctomedicina.

Comentarios de Test a distancia 1. vuelta

Psiquiatra
En los nios, los sndromes depresivos van a ocultarse tras quejas fsicas (los
nios pequeos carecen de lenguaje emocional y tienden a expresarse
somticamente), trastornos de conducta o descenso del rendimiento
acadmico; en los adolescentes, los trastornos de conducta, incluido
el abuso de sustancias, pueden enmascarar una depresin. No hay que
olvidar que, en nios y adolescentes el estado de nimo predominante
durante una depresin va a ser con frecuencia irritable ms que triste,
lo que dicultar el diagnstico.
Pregunta 19.-R: 3
Dentro de los sndromes depresivos, los sntomas fsicos son los ms fciles
de diagnosticar, pues tienen un carcter objetivo; sin embargo, carecen de
especicidad diagnstica al presentarse tambin en casi todas las enfermedades psiquitricas y mdicas; por sto, en los pacientes con enfermedades
psiquitricas o mdicas coexistentes, habr que recurrir a sntomas ms
subjetivos (estado de nimo, pensamientos) para asegurar el diagnstico.
La alteracin del sueo ms frecuente en una depresin es el insomnio de
caractersticas inespeccas (problemas de conciliacin con despertares
nocturnos); cuando aparece tendencia al despertar precoz sugiere una
depresin melanclica (EDM con sntomas melanclicos o depresin
endgena), sobre todo si se asocia con una variacin diurna del estado de
nimo en la forma de empeoramiento matutino y mejora vespertina. Los
pacientes con sntomas atpicos suelen referir una desagradable necesidad
de dormir durante el da, quejndose adems de un cansancio extremo.
En lo que hace referencia al apetito y al peso, lo ms frecuente es ver una
disminucin de ambos; en los pacientes atpicos habr aumento del apetito y del peso, describindose una curiosa tendencia a comer mayores
cantidades de hidratos de carbono (dulces) de lo habitual para el paciente.
Pregunta 20.-R: 4
En muchos pacientes depresivos el estado de nimo ucta en funcin de
diversos factores. Solamente en las depresiones ms graves el estado de
nimo permanece inalterable todo el da, con una ausencia de reactividad
ante lo que le rodea (anestesia emocional). En los sndromes depresivos
leves los pacientes conservan cierta capacidad de mejorar cuando las
circunstancias externas les son favorables (se dice que preservan la
reactividad); as tienden a notarse mejor cuanto ms activos estn o
cuando estn acompaados; como en general estamos ms ocupados
por las maanas, puede dar la impresin de que estos pacientes siguen
un ritmo circadiano, lo que biolgicamente no es cierto.
En los cuadros melanclicos el paciente puede encontrarse algo mejor
por la noche (se conoce como mejora vespertina), pero lo que s va a
referir con frecuencia es que est mucho peor por la maana, nada ms
despertarse (empeoramiento matutino); la asociacin entre despertar
precoz y empeoramiento matutino es altamente especca de melancola
(o depresin endgena), y se ha relacionado con alteraciones en los
marcapasos endgenos pues sigue un ritmo circadiano muy parecido al
ritmo del cortisol (que con frecuencia est alterado en estos pacientes).
La respuesta selectiva a los IMAO frente a tricclicos es una de las caractersticas de las depresiones atpicas.
La asociacin entre depresin y estaciones no es tan frecuente como se
cree popularmente; tan slo un 15% de los pacientes con enfermedades
afectivas recurrentes (depresivas o bipolares) presentan un ritmo estacional claro, que se repite de ao en ao. Lo habitual en estos casos es que
los EDM aparezcan en otoo y los episodios manacos en verano. En los
pases nrdicos se ha descrito una variacin del ritmo estacional en los
pacientes bipolares con EDM en invierno (frecuentemente con sntomas
atpicos) y episodios manacos en verano. Este cambio se ha puesto en
relacin con la duracin del fotoperodo (muy corta en invierno, muy
larga en verano), proponindose el tratamiento con fototerapia con
carcter preventivo o curativo.
CTO Medicina C/Francisco Silvela, 106 28002 - Madrid

Pregunta 21.-R: 2
Uno de los problemas ms frecuentes ante el diagnstico de depresin
es la tendencia casi automtica a buscar un factor psicosocial estresante
que consideramos desencadenante. Para empezar, no hay que olvidar
que muchas personas se enfrentan a diario a situaciones estresantes muy
similares, pero slo una parte de ellas desarrollan un sndrome depresivo;
hay una serie de factores individuales que aumentan el riesgo de presentar una depresin, entre los que destaca la presencia de antecedentes
familiares, los antecedentes personales de depresin y determinados
rasgos de personalidad. Los factores estresantes parece que actan
de una forma inespecca, aumentando la probabilidad de presentar
un sndrome psiquitrico en funcin de la predisposicin individual.
Adems, no existe una buena correlacin entre la presencia o ausencia
de un factor estresante y la duracin, la intensidad o las caractersticas
del sndrome depresivo asociado; la mitad de los pacientes con EDM
no reeren acontecimiento estresante alguno en las semanas previas
al inicio de su depresin, y su evolucin no es diferente de la evolucin
de los pacientes que s reeren un acontecimiento causal.
Hoy en da se reserva el trmino trastorno adaptativo con sntomas
depresivos para aquellos pacientes que presentan sntomas depresivos
de intensidad leve en respuesta a una situacin estresante que produce
reacciones similares en otras personas; estos sntomas se resuelven
espontneamente en muchos casos y no suelen precisar tratamiento
farmacolgico, salvo que su intensidad aumente.
El trmino depresin reactiva engloba, por tanto, desde episodios
depresivos mayores, a distimias y trastornos adaptativos.
Pregunta 22.-R: 3

Pregunta 22. Relacin entre la intensidad de la depresin


y la modalidad de tratamiento.
En los sndromes afectivos ms graves los pacientes pueden presentar
sntomas psicticos; en general estos sntomas son ms probables en
los cuadros manacos que en los depresivos; los delirios son mucho
ms frecuentes que las alucinaciones, por lo que usamos los trminos
depresin (o mana) psictica o delirante de forma similar.
En la mayora de los pacientes con estos sntomas encontramos una
relacin lgica entre el contenido de los delirios y alucinaciones y su
estado de nimo (sntomas psicticos congruentes, ideas deliroides o
ideas delirantes secundarias); si estn deprimidos, su visin pesimista
se extiende hacia el pasado (ideas de culpa), el presente (ideas de minusvala y enfermedad) y el futuro (ideas de ruina); si estn manacos,
su optimismo y autoconanza puede llegar a los delirios de grandeza.
En algunos casos, los delirios y alucinaciones se alejan de los contenidos

Tfno. (0034) 91 782 43 30/33/34 E-mail: secretaria@ctomedicina.com www. ctomedicina.com

Comentarios de Test a distancia 1. vuelta

Psiquiatra
lgicos (delirios de perjuicio, delirios de control), planteando el diagnstico diferencial con la esquizofrenia y los trastornos esquizoafectivos; no
hay un acuerdo acerca de la importancia de estos sntomas psicticos
incongruentes con el estado de nimo, variando la forma de clasicacin
de estos pacientes desde su inclusin en el trastorno esquizoafectivo
(CIE) hasta el mantenimiento del diagnstico de trastorno afectivo (DSM).
Pregunta 23.-R: 3
La respuesta habitual de un paciente deprimido al tratamiento con
antidepresivos es la vuelta a la normalidad. De hecho, si una persona
que no est deprimida toma estos medicamentos NO va a notar un aumento del estado de nimo, sino simplemente efectos secundarios (los
antidepresivos NO son euforizantes y los euforizantes, como la cocana,
NO tienen efecto antidepresivo).
En algunos casos los pacientes sufren un cambio radical de su estado
de nimo en respuesta al tratamiento, alcanzando una intensidad hipomanaca o manaca a gran velocidad. Esta respuesta tan rpida es tpica
de los pacientes bipolares, recomendndose en ellos la retirada del frmaco antidepresivo en cuanto se consigue la normalizacin para evitar
el cambio de fase (a diferencia de los pacientes depresivos, en los que
se mantiene el mismo frmaco 6 meses a la misma dosis). Sin embargo,
vemos pacientes que tienen respuestas maniformes al tratamiento con
antidepresivos sin que hayan presentado nunca episodios manacos
espontneos; ocialmente no se pueden catalogar como bipolares,
pero debera tenerse en cuenta en ellos la posibilidad de tratamiento
de prevencin de recadas con litio.
Pregunta 24.-R: 5
A mediados del siglo XX se pudo diferenciar la enfermedad bipolar de la
enfermedad depresiva atendiendo a la mayor frecuencia de antecedentes
familiares afectivos en la primera; de hecho, hay autores que ante un
paciente depresivo con antecedentes familiares de enfermedad bipolar
proponen ya el tratamiento prolctico con litio, entendiendo que ser
cuestin de tiempo el que presente un episodio maniforme.

FORMAS UNIPOLARES

Otras diferencias entre la enfermedad bipolar y la depresiva son de tipo


epidemiolgico (predominio de mujeres en la enfermedad depresiva,
inicio ms temprano en la enfermedad bipolar, mayor riesgo [relativo]
de suicidio en la enfermedad bipolar). Se han realizado numerosas
comparaciones entre las recadas depresivas de ambas enfermedades;
parece que las depresiones bipolares tienden a ser algo ms cortas que las
unipolares (6 meses frente a 12 meses), aunque con una mayor frecuencia
de recidivas a lo largo de la vida (6-9 frente a 2-3). En general, los sntomas
son muy parecidos, aunque se describen ms sntomas atpicos en las
formas bipolares (sobre todo en las bipolares-2) y una mayor tendencia
a la agitacin psicomotora en las depresiones unipolares. No hay que
olvidar que en la enfermedad bipolar podemos ver cualquier variante
de EDM (melanclico, atpico, psictico, catatnico) y diferentes formas
de episodios manacos (mana clsica eufrica, mana atpica disfrica,
hipomana, episodios mixtos).
Pregunta 25.-R: 3
La mayora de los pacientes bipolares debutan con un EDM, siendo
de especial inters la posibilidad de predecir una evolucin bipolar desde el primer episodio. El principal factor de riesgo para ser
bipolar es tener antecedentes familiares de esa enfermedad, dado
que la enfermedad bipolar tiene la heredabilidad ms alta de toda
la psiquiatra (cercana al 90%). En las mujeres, un EDM puerperal
evolucionar hacia una enfermedad bipolar en cerca del 80% de los
casos. El debut en la adolescencia tiende a ser ms frecuente en las
formas bipolares (pero NO es determinante). La respuesta anormalmente rpida al tratamiento antidepresivo y la aparicin de cuadros
maniformes secundarios al mismo, son tambin dos caractersticas
que sugieren bipolaridad.
Cuando los pacientes debutan con un episodio manaco, el diagnstico
es inmediato; los varones tienen una mayor tendencia a presentar este
tipo de comienzo y a tener ms fases manacas, lo que con frecuencia
plantea problemas de diagnstico con la esquizofrenia (varones, jvenes, conducta desorganizada y agitada, delirios y alucinaciones) y los
comportamientos antisociales (varones, jvenes, conductas arriesgadas,
abuso de sustancias).

FORMAS BIPOLARES

Prevalencia-vida

Alta (15%)

Baja (1%)

Distribucin sexual

Mujeres > hombres

Mujeres = hombres

Edad de inicio

Tarda (> 40 aos)

Joven (< 30 aos)

Clase social

Baja?

Alta?

Personalidad previa

Sana
(rasgos melanclicos)

Sana
(rasgos ciclotmicos)

Antecedentes
familiares

Frecuentes (unipolares)

Muy frecuentes
(bipolares, unipolares)

Recadas

Pocas (1-3)

Muchas (6-9)

Duracin del episodio depresivo

Larga (12-24 meses)

Corta (6-9 meses)

Alteraciones
psicomotoras
en la depresin

Agitacin

Inhibicin

Riesgo de suicidio

Menor

Mayor

Induccin de mana/hipomana

No

Prevencin de
recadas

Antidepresivos

Estabilizadores

Pregunta 24. Comparacin entre la depresin recurrente


y el trastorno bipolar.
CTO Medicina C/Francisco Silvela, 106 28002 - Madrid

Pregunta 26.-R: 3
Actualmente se aceptan dos variantes principales de trastorno afectivo
bipolar (TAB). En el TAB-1, los pacientes tienen episodios manacos o
mixtos, asociados casi siempre a episodios depresivos; como se ve,
cabe la posibilidad de que un paciente tenga slo episodios manacos
o mixtos, pero es algo excepcional y no parece que evolucionen de
forma diferente a los pacientes manaco-depresivos.
En el TAB-2 veremos EDM junto a episodios hipomanacos; estas
hipomanas son difciles de diagnosticar y no es raro que estos
pacientes se clasifiquen errneamente como depresivos, privndoles del tratamiento con estabilizadores, lo que explica su peor
pronstico a largo plazo (mayores tasas de suicidio, ms tendencia
a la ciclacin rpida); en los pacientes con TAB-2 vemos un ligero
predominio femenino, mientras que en los TAB-1 hay un equilibrio
entre sexos.
Se acepta slo una forma menor de trastorno bipolar, la ciclotimia, denida por la alternancia de sntomas hipomanacos y sntomas depresivos
leves durante un perodo mnimo de 2 aos.
La alternancia de episodios depresivos invernales y episodios maniformes
en verano caracteriza a la forma estacional (nrdica) del TAB que suele
tratarse de un TAB-2 (EDM ms hipomana).
La aparicin de un EDM sobre una distimia previa se conoce a veces como
depresin doble. Implica una mayor gravedad que los EDM aislados, con
menor tendencia a la recuperacin.

Tfno. (0034) 91 782 43 30/33/34 E-mail: secretaria@ctomedicina.com www. ctomedicina.

Comentarios de Test a distancia 1. vuelta

Psiquiatra
y de intento de suicidio para referirse a los pacientes que se autolesionan
por otras razones.
Pregunta 28.-R: 2
Se llama depresin secundaria o exgena a todo sndrome depresivo
para el que se encuentra una causa mdica, txica o farmacolgica.
En la prctica destacaremos tres causas especialmente frecuentes:
Frmacos y drogas: aquellos frmacos que alteran la funcin noradrenrgica central (betabloqueantes, reserpina, alfa-metil-dopa, clonidina), frmacos con actividad esteroidea (estrgenos, progestgenos,
corticoides, ACTH), intoxicacin crnica por drogas sedantes (alcohol,
opiceos, benzodiacepinas) y abstinencia de drogas estimulantes
(cocana, anfetaminas).
Enfermedades endocrinolgicas: sobre todo el hipotiroidismo, pero
tambin el hipertiroidismo (sobre todo en ancianos), el hiperparatiroidismo o los trastornos adrenales (Cushing, Addison).
Enfermedades neurolgicas: tanto las enfermedades degenerativas
que afectan de forma ms o menos difusa a todo el sistema nervioso
central (Alzheimer, Parkinson) como algunas enfermedades localizadas (tumores, accidentes vasculares, abscesos) que lesionan sobre
todo las regiones frontales.

Pregunta 26. Espectro bipolar.


Pregunta 27.-R: 4
En psiquiatra diferenciamos dos conceptos referidos al suicidio que
popularmente se mezclan de forma inadecuada.
En primer lugar hablamos de SUICIDIO cuando una persona se autolesiona de forma voluntaria y deliberada con la intencin de acabar con
su vida. Por otra parte, hablamos de AUTOLESIN (sin ms) cuando una
persona se provoca algn tipo de dao fsico sin que existiera un deseo
explcito de muerte. Estas autolesiones son mucho ms frecuentes que
los verdaderos intentos de suicidio (100 veces o ms) y, en general, se
pueden dividir en aqullas que tienen una nalidad MANIPULADORA
(distraer o llamar la atencin del entorno, conseguir algn tipo de
benecio) y aqullas que tienen un carcter ms IMPULSIVO (como las
reacciones en cortocircuito de los pacientes con personalidad lmite).
Ante cualquier autolesin no accidental debe solicitarse una valoracin
psiquitrica que determinar la intencionalidad de la misma. La mayora
de las autolesiones no suicidas se presentan en jvenes, sobre todo mujeres, que se enfrentan a una situacin estresante ms o menos grave,
careciendo de otros recursos psicolgicos para abordarla (con frecuencia
hay rasgos patolgicos de personalidad); suelen elegirse mtodos de baja
letalidad (intoxicacin con medicamentos, cortes con armas blancas) y
no se prepara el acto con antelacin, siendo relativamente fcil el rescate
(a veces se llevan a cabo en presencia de la propia familia); tras su atencin mdica en Urgencias y la valoracin psiquitrica se les remite a los
dispositivos ambulatorios, siendo excepcional la necesidad de ingreso.
En cambio, la mayora de los intentos de suicidio se dan en personas mayores de 50 aos (una excepcin notable son los intentos de
suicidio que presentan los jvenes con esquizofrenia), varones que
sufren enfermedades psiquitricas bien denidas (depresin, abuso de
sustancias, esquizofrenia), asociadas o no a otros factores adversos de
tipo mdico (enfermedades crnicas, dolorosas, incapacitantes) o social
(viven solos, han perdido una relacin signicativa recientemente, pasan
por dicultades econmicas); suelen utilizarse mtodos ms agresivos
(precipitacin desde gran altura, ahorcamiento, armas de fuego) y
se planica mejor el acto, evitando ser descubiertos; si sobreviven al
intento de suicidio, suelen ingresar y es necesario adoptar una serie
de precauciones para que no vuelvan a intentar suicidarse durante el
ingreso (sedacin, contencin mecnica, vigilancia constante).
Sin embargo, conviene tener en cuenta que algunos libros hablan de
suicidio consumado para referirse a los pacientes autnticamente suicidas
CTO Medicina C/Francisco Silvela, 106 28002 - Madrid

La isoniacida tiene actividad IMAO y fue uno de los primeros antidepresivos usados en psiquiatra a principios de los aos 50, siendo
posteriormente sustituido por otros frmacos con menor toxicidad; por
tanto, comparte con ellos la capacidad de producir cuadros maniformes.
Pregunta 29.-R: 5
La utilidad de los antidepresivos va mucho ms all de los sndromes
depresivos, de ah que en la actualidad se preera denominarles por
su mecanismo de accin (ISRS, IMAO, inhibidores no selectivos de la
recaptacin).
Dentro de los trastornos psiquitricos van a usarse antidepresivos en casi
todos los trastornos por ansiedad (en el pnico para prevenir las crisis, en
la fobia social grave, en el trastorno obsesivo-compulsivo, en el trastorno
por estrs postraumtico y en el trastorno por ansiedad generalizada).
Tambin los utilizamos para frenar la impulsividad en pacientes con trastornos de la conducta alimentaria (atracones bulmicos), trastornos del
control de los impulsos (juego patolgico o ludopata), personalidades
tipo lmite o trastornos por abuso de sustancias (alcoholismo). Dentro
de estos ltimos pacientes, los antidepresivos van a tener cierta capacidad de aliviar el sndrome de abstinencia de la cocana y parece que
protegen de la neurotoxicidad inducida por xtasis y otras anfetaminas
de diseo. En algunos trastornos psiquitricos propios de la infancia
(enuresis, trastorno por dcit de atencin) pueden utilizarse, aunque
no sean el tratamiento de primera eleccin. Por la capacidad sedante
de algunos antidepresivos se proponen stos como alternativa a las
benzodiacepinas en algunos casos de insomnio.
Ya en un terreno ms neuropsiquitrico, vemos cmo los antidepresivos
tricclicos se utilizan para la prevencin de la cefalea tensional y el tratamiento de dolores neuropticos crnicos. Tienen adems la capacidad
de reducir la frecuencia de episodios de cataplejia en la narcolepsia. El
efecto antihistamnico de la doxepina supera a la mayora de los antihistamnicos ociales, y no es raro que se use para el tratamiento del
prurito idioptico.
Pregunta 30.-R: 3
A la hora de enfrentarnos al tratamiento de un sndrome depresivo,
vamos a decidirnos por un tipo de tratamiento u otro en funcin de la
intensidad de los sntomas. Cuando un sndrome depresivo se acerca a

Tfno. (0034) 91 782 43 30/33/34 E-mail: secretaria@ctomedicina.com www. ctomedicina.com

Comentarios de Test a distancia 1. vuelta

Psiquiatra
la intensidad de EDM, la probabilidad de respuesta a frmacos antidepresivos (AD) supera el 60%, siendo mayor an en los casos de EDM con
sntomas melanclicos. En las formas extremas de depresin (depresin
psictica) ser necesario combinar AD con antipsicticos o recurrir al
tratamiento con electrochoque. En las formas menores de depresin la
respuesta a frmacos ronda el 40% y se acerca mucho a la alcanzada con
placebo, logrndose como mucho un alivio sintomtico.
Una vez decidida la necesidad de poner un AD, la eleccin entre los
distintos compuestos depender de:
Los antecedentes de respuesta a frmacos en episodios previos.
La presencia de otras enfermedades que contraindiquen el uso de
un AD en concreto (por ejemplo, una cardiopata contraindicara el
uso de AD tricclicos).
La toma de alguna medicacin que interaccione con el AD.
El perl de efectos adversos del AD que puedan provocar su rechazo
por el paciente.
Los antecedentes de respuesta a un AD en concreto en un familiar
de primer grado.
El patrn de sntomas depresivos (por ejemplo, los sntomas atpicos
obligan a usar IMAO).
La comodidad de su uso.
El precio del frmaco.
Teniendo en cuenta que la mayora de los pacientes se presentan sin
haber tomado nunca medicacin AD o no la recuerdan, los ISRS renen
las caractersticas idneas para convertirse en los AD de primera eleccin.
Pregunta 31.-R: 3
Los antidepresivos no mejoran la depresin rpidamente, si no que tienen
un efecto a medio plazo, tardando varias semanas (4-6) en conseguir la
remisin de los sntomas, aunque previamente ya se hayan detectado
signos de respuesta.
Una vez alcanzada la remisin, se propone la necesidad de continuar
con el mismo tratamiento (el mismo AD, a la misma dosis) un mnimo
de 6 meses para prevenir la recada del episodio.
Pasado ese tiempo (que dependiendo de diversos factores, como la
mayor edad, la gravedad del episodio o el nmero de episodios previos,
podr alargarse), se considera que el paciente se ha recuperado del
episodio depresivo. Entonces se podr retirar el AD (casi siempre de
forma progresiva, en unas semanas para minimizar posibles sntomas
abstinenciales), salvo a los pacientes con alto riesgo de recurrencia
(como aqullos que hayan sufrido numerosos episodios), a los que se
les propondr tratamiento indenido de mantenimiento (que suele
realizarse en la dosis mnima posible para reducir los efectos adversos
y mejorar as el cumplimiento).

Pregunta 31. Fases del tratamiento de la depresin.


CTO Medicina C/Francisco Silvela, 106 28002 - Madrid

Con los modernos ISRS todo este proceso de tratamiento se simplica,


pues la dosis inicial suele ser la dosis suciente (no hay que subir poco
a poco, para que el paciente se acostumbre a los efectos adversos como
con los AD tricclicos), tanto para el tratamiento agudo y de continuacin,
como para el tratamiento de mantenimiento a largo plazo.
Se discute si alguno de los AD ms modernos (venlafaxina o mirtazapina) tendra un efecto algo ms rpido aunque las diferencias no son
clnicamente relevantes.
Pregunta 32.-R: 5
Los AD tricclicos (ADT) son los AD ms antiguos y, por tanto, aqullos
de los que se dispone de ms experiencia. De hecho, la imipramina
se considera el AD patrn con el que deben compararse los dems
antidepresivos. Los ADT logran su efecto antidepresivo aumentando la
disponibilidad de las principales monoaminas (noradrenalina, serotonina
y dopamina) al inhibir su recaptacin.
Sin embargo, no lo hacen de una forma selectiva, sino que tambin
bloquean numerosos receptores, cada uno de los cuales va a producir
una serie de efectos secundarios. No todos los ADT son iguales en esta
caracterstica; los hay con potentes efectos anticolinrgicos (amitriptilina,
clorimipramina) y los hay con menos problemas a ese nivel (nortriptilina,
desipramina); estos efectos muscarnicos son muy mal tolerados, especialmente por pacientes ancianos. Asimismo, tenemos ADT muy sedantes
por su efecto antihistamnico (amitriptilina, doxepina, clorimipramina)
y otros ms activadores con escasa capacidad sedante (imipramina).
Lo que suele ser necesario en todos los ADT es comenzar con dosis
muy bajas e ir subiendo poco a poco, para lograr cierta tolerancia a los
efectos adversos. Un problema adicional de los ADT son sus efectos
cardiovasculares; por su capacidad de bloqueo adrenrgico pueden
causar hipotensin ortosttica y por su accin parecida a la quinidina
sobre la conduccin cardaca cabe la posibilidad de que produzcan
arritmias potencialmente letales. La intoxicacin con ADT supone un
serio problema en los pacientes depresivos con ideas suicidas.
Sin embargo, todava se utilizarn atendiendo a tres ventajas:
Su precio (7-10 veces inferior al precio de los nuevos AD), lo que
resulta signicativo en pases con menor nivel de desarrollo.
La posibilidad de controlar el cumplimiento mediante los niveles
plasmticos (sobre todo con imipramina).
Su posible mayor potencia antidepresiva (que se explicara por el
hecho de que la mayora de los ADT actan a un doble nivel: noradrenrgico y serotoninrgico).
En realidad, salvo la imipramina en depresiones resistentes a otros AD,
la amitriptilina en el control del dolor crnico y la clorimipramina en el
TOC, el resto de ADT se usan de forma anecdctica.

Pregunta 32. Esquema de los efectos de los antidepresivos tricclicos


y posibles efectos secundarios.

Tfno. (0034) 91 782 43 30/33/34 E-mail: secretaria@ctomedicina.com www. ctomedicina.

Comentarios de Test a distancia 1. vuelta

Psiquiatra
Pregunta 33.-R: 1
Al principio de la dcada de los aos 80, los ISRS irrumpen en el mercado
psicofarmacolgico con tres ventajas evidentes:
Un perl de efectos secundarios mucho ms tolerable.
Una mayor comodidad de uso, al no tener que aumentar lentamente
la dosis, ni fraccionarla en varias tomas diarias.
La casi ausencia de letalidad en el caso de sobredosis.
En apenas 10 aos desplazan casi por completo a los ADT y a los IMAO
en el terreno de la depresin, y poco a poco van conquistando otras
indicaciones (pnico, trastorno obsesivo, bulimia, fobia social) hasta
alcanzar actualmente el 90% de las prescripciones de antidepresivos en
Espaa. Se consideran frmacos de primera eleccin en el tratamiento de
la depresin (depresin mayor, distimia) y de los trastornos de ansiedad.
Sin embargo, tienen tres aspectos discutibles:
Su precio es superior al de los antidepresivos tricclicos, aunque los
estudios de coste-efectividad han demostrado que la diferencia de
precio del frmaco se compensa por otros costes relacionados con
la depresin (das de baja laboral, necesidad de ingreso, necesidad
de medicacin adicional para tratar efectos adversos). Adems, la
aparicin de innumerables genricos ha bajado mucho su precio.
Se discute si tienen una menor potencia antidepresiva (debida quizs a
su efecto exclusivamente serotoninrgico), aunque los estudios sobre
efectividad en condiciones reales (no de ensayo clnico) demuestran
que los pacientes tienden a abandonar el tratamiento con ADT o no
alcanzan dosis adecuadas por culpa de los efectos adversos, con lo
que la terica mayor ecacia de los ADT se pierde, igualndose su
efectividad a la de los ISRS.
Van a presentar numerosas interacciones farmacocinticas en los
citocromos P-450 hepticos, a los que inhiben en mayor o menor
medida (parece que citalopram y sertralina menos que los dems
ISRS) modicando al alza los niveles plasmticos de diversos frmacos,
algunos potencialmente peligrosos por sus efectos cardiotxicos.
Antidepresivo

Ventajas

Desventajas

ADT

- Ecacia antidepresiva
demostrada
- Precio
- Control de niveles
plasmticos (imipramina)

- Numerosos efectos
secundarios
- Letalidad en sobredosis
- Posologa compleja
(ascenso, fraccionamiento)

ISRS

- Escasos efectos
secundarios
- Escasa letalidad
en sobredosis
- Comodidad de uso

- Precio elevado
- Interacciones
farmacocinticas
- Ecacia antidepresiva
menor?

Pregunta 33. Ventajas y desventajas de los antidepresivos tricclicos y los


inhibidores selectivos.
Pregunta 34.-R: 2
Los IMAO tenan un perl de tolerancia ligeramente mejor que los ADT,
pero dos importantes problemas prcticos:
La necesidad de seguir una dieta para evitar posibles crisis hipertensivas adrenrgicas secundarias a compuestos vasopresores
derivados de las aminas (tiramina).
Algunas interacciones farmacolgicas potencialmente peligrosas
por sus efectos noradrenrgicos (vasoconstrictores nasales) o serotoninrgicos (ISRS, meperidina).
Aunque la dieta no es complicada (bsicamente se trata de evitar todos los
alimentos que han sufrido procesos de fermentacin, como el queso, las
bebidas alcohlicas fermentadas como el vino o la cerveza, los ahumados,
CTO Medicina C/Francisco Silvela, 106 28002 - Madrid

las conservas y los salazones) lentamente se han ido abandonando; en la


actualidad slo nos queda un IMAO clsico en Espaa (tranilcipromina),
aunque en otros pases (Francia, Inglaterra, EE.UU.) an podemos encontrar otros IMAO (la fenelcina es el ms usado en el mundo).
Estrictamente hablando, habra slo una indicacin para el uso de
IMAO como primera opcin: la depresin con sntomas atpicos.
Sin embargo, incluso en estos casos es cada vez ms frecuente el
uso de ISRS.
Las alternativas a los IMAO clsicos son los IMAO reversibles de la MAOA (como la moclobemida) que lamentablemente tienen una potencia
antidepresiva muy baja y los IMAO selectivos de la MAO-B (como la
selegilina o la rasagilina) que se usan para la enfermedad de Parkinson
pues prcticamente slo consiguen elevar la dopamina.
Pregunta 35.-R: 3
La terapia electroconvulsiva (TEC) fue la primera estrategia realmente
ecaz para el tratamiento de la depresin; se comenz a utilizar a nales
de los aos 30, y hasta la revolucin farmacolgica de los aos 50-60
fue el instrumento preferido para el tratamiento agudo de la depresin.
Tras unos aos de rechazo, ms por motivos ideolgicos que por razones cientcas, desde el nal de la dcada de los aos 80 asistimos a un
renacimiento de la TEC. La TEC tiene tres ventajas sobre los frmacos AD:
Es ms potente en su accin antidepresiva, lo que explica su principal
indicacin, la depresin mayor resistente a AD, y el que, en ocasiones,
se plantee su uso como primera opcin en la forma ms grave de
depresin, la depresin psictica o delirante.
Logra su efecto antidepresivo ms rpido que los frmacos (en 2-3
semanas cuando se aplica a das alternos), lo que explica su utilidad
en las depresiones estuporosas o catatnicas (con graves problemas
mdicos asociados) y en las depresiones con intensas ideas suicidas.
Tiene escasos efectos secundarios fsicos, lo que explica su utilidad en
casos de intolerancia o contraindicacin para el uso de antidepresivos,
en pacientes polimedicados o con patologa orgnica compleja y en
las psicosis graves durante el embarazo.
Las guas clnicas llegan a decir que no existe ninguna contraindicacin absoluta para la TEC pero algunos libros mencionan el riesgo que
supone utilizarla en casos de hipertensin intracraneal (pues durante
cualquier crisis epilptica aumenta la produccin de lquido cefalorraqudeo y cabra la posibilidad de una herniacin cerebral). En general,
las contraindicaciones dependen del procedimiento anestsico y cada
vez son ms raras.
El principal efecto secundario es de tipo neuropsicolgico: la produccin
de una amnesia antergrada. Depende del estado cognitivo previo
(ms frecuente en ancianos), del nmero de sesiones recibidas, de
la carga elctrica suministrada y de la disposicin de los electrodos.
Al cabo de unos meses NO existen secuelas del tratamiento (salvo
ciertas lagunas de memoria consecuencia del perodo de amnesia)
y el funcionamiento cognitivo de los pacientes es incluso mejor que
si hubieran recibido frmacos, pues la recuperacin de la depresin
llega a ser ms completa.
La mala fama de la TEC procede de la poca inicial, en la que se aplicaba
sin anestesia (por estar poco desarrollados los procedimientos anestsicos) y en la que se usaba de forma indiscriminada para casi cualquier
tipo de paciente psiquitrico.
No hay que olvidar que fuera de la depresin mayor la TEC apenas tiene
indicaciones; se puede usar en los episodios manacos, cuando son muy
graves o se muestran resistentes a los frmacos (algo poco frecuente);
tambin se va a utilizar en episodios esquizofrnicos agudos resistentes a
antipsicticos, sobre todo si existen sntomas catatnicos o si hay clnica
depresiva asociada.

Tfno. (0034) 91 782 43 30/33/34 E-mail: secretaria@ctomedicina.com www. ctomedicina.com

10

Comentarios de Test a distancia 1. vuelta

Psiquiatra
Pregunta 36.-R: 4
El litio es el estabilizador de primera eleccin en la enfermedad bipolar.
Su principal benecio es la reduccin del nmero de recadas (sobre
todo manacas) y tambin consigue atenuar la intensidad de las mismas,
caso de que se produzcan.
Ms discutible es su ecacia en los episodios agudos; como antidepresivo, el litio es muy poco potente y no es raro que se deban usar otros
frmacos asociados (antidepresivos con muchas precauciones por el
riesgo de cambio de fase: lamotrigina, quetiapina) o recurrir al TEC;
como antimanaco, el litio es muy ecaz pero tiene un efecto retardado
(varias semanas) que puede resultar inaceptable para pacientes manacos
graves y, adems, no se dispone de otra presentacin aparte de la oral,
lo que condiciona su uso a la aceptacin del tratamiento por parte del
paciente (que no es lo habitual en las formas graves); por eso, en las
fases manacas graves se recurre al uso de frmacos por va parenteral
(antipsicticos, benzodiacepinas), para introducir el estabilizador en un
segundo momento. El litio es especialmente ms ecaz en los cuadros de
mana clsica (eufrica), disminuyendo su ecacia en los cuadros atpicos
(manas disfricas o irritables, fases mixtas, etc.), prerindose en estos
casos otros frmacos (valproico, carbamacepina, antipsicticos atpicos).
Adems, el litio es un medicamento con numerosos efectos secundarios,
hasta tal punto que si un paciente no reere ningn problema durante
el tratamiento, probablemente no lo est tomando. Para prevenir los
efectos adversos ms graves se realizan una serie de controles mdicos
antes y durante el tratamiento con litio; con diferencia, lo ms importante
ser el control de los niveles plasmticos de litio (litemia), pues guardan
una relacin bastante estrecha con la ecacia y la toxicidad.

Hemograma
Pruebas de funcin renal (creatinina, urea)
Estudio inico (sodio, potasio, calcio)
Pruebas de funcin tiroidea (TSH, T4 libre)
ECG
Test de embarazo (al inicio)
Glucemia y cuerpos cetnicos (si se sospecha intolerancia a los hidratos
de carbono)
Pruebas de concentracin de la orina (si se sospecha diabetes inspida
nefrognica)

Pregunta 36. Controles necesarios en el tratamiento con litio.


Dado que el litio se elimina por el rin, utilizando los mismos sistemas
de transporte que el sodio, ser necesario disponer de parmetros que
nos informen del estado de la funcin renal (creatinina, BUN).
Uno de los efectos ms peligrosos es la alteracin de la conduccin
cardaca; las alteraciones en el ECG permiten controlar hasta qu punto
el litio produce problemas a este nivel (normalmente se van a limitar a
cambios en la onda T).
El litio es un medicamento teratgeno; se ha relacionado con malformaciones cardacas (anomala de Ebstein), por lo que antes de iniciar el tratamiento con litio en una mujer en edad frtil conviene asegurarse de que no
est embarazada e informarla de la necesidad de que adopte medidas de
anticoncepcin ecaces; ante cualquier sospecha de embarazo habr que
repetir la prueba diagnstica de gestacin pues, de conrmarse, se debera
interrumpir el tratamiento (nunca de forma brusca por el riesgo de recada).
Finalmente, resulta obligatorio el control de la funcin tiroidea mediante
la determinacin de la TSH para detectar problemas tiroideos (que en
ocasiones explican la resistencia al tratamiento). Otras determinaciones
(glucemia, calcemia, pruebas de concentracin de la orina) dependern
de la evolucin y de la aparicin de efectos secundarios.
El electroencefalograma NO es una prueba necesaria para el control del
litio ni de ningn otro psicofrmaco; cuando aparecen alteraciones en
CTO Medicina C/Francisco Silvela, 106 28002 - Madrid

el EEG es que ya se ha provocado neurotoxicidad, que con frecuencia


ser irreversible.
Pregunta 37.-R: 5
Los niveles de litio deben estar entre 0,4 y 1,5 mEq /l. Lgicamente, en
los episodios agudos sern necesarias litemias ms altas (superiores a
1,0 mEq/l) que en los perodos de estabilidad (inferiores a 1,0 mEq/l); la
mayora de los pacientes van a estar un poco por arriba o por debajo de
esta cifra (1,0 mEq/l), para no provocar una excesiva toxicidad o perder
la ecacia.

Pregunta 37. Mrgenes de las litemias.


Si el episodio manaco o depresivo es leve, quizs baste con una ligera
correccin de la dosis del estabilizador, pero en los dems casos ser
necesario aadir tratamiento especco; en los casos de mana grave la
falta de colaboracin del paciente suele obligar al tratamiento inicial con
psicofrmacos parenterales (antipsicticos, benzodiacepinas), dejando
la adicin del estabilizador (oral) para ms adelante.
Una vez conseguido el control de la recada se retirar el frmaco especco
de la fase en cuanto sea posible, para no inducir un cambio de polaridad.
Cerca de 1/3 de los pacientes podrn estabilizarse a largo plazo con un
solo frmaco, pero cerca de un 20% de los pacientes precisarn una pauta
de medicacin compleja (con cuatro o ms frmacos).
Pregunta 38.-R: 4
Otro problema adicional del litio es la posibilidad de que los niveles
plasmticos se modiquen por factores que afectan la eliminacin renal,
sin que el paciente haya variado la dosis. Dado que el litio utiliza los
mismos sistemas de transporte renal que el sodio (se reabsorbe sobre
todo en el tbulo proximal), cualquier circunstancia que aumente la
reabsorcin de este ion (hipovolemia, deshidratacin, hiponatremia)
producir un aumento de los niveles de litio, superando el umbral de
toxicidad. Numerosos frmacos van a producir los mismos problemas,
bien por sus efectos sobre la vascularizacin renal (AINE, inhibidores
de la COX-2), bien por sus efectos sobre el tbulo renal (diurticos,
IECA, ARA-2).
En teora podramos ver una disminucin de la litemia, con la consiguiente
prdida de ecacia, en las situaciones hidroelectrolticas inversas (hipernatremia, ingesta hdrica excesiva) y por algunos frmacos (metilxantinas,
diurticos osmticos e inhibidores de la anhidrasa carbnica), pero son
situaciones excepcionales.
Pregunta 39.-R: 2
La ciclacin rpida es una complicacin extremadamente grave en la
evolucin del trastorno bipolar. El paciente pasa de una fase a otra casi
sin estar unos das estable. Sucede con ms frecuencia en bipolares-2,
muchas veces no diagnosticados de bipolares y tratados exclusivamente

Tfno. (0034) 91 782 43 30/33/34 E-mail: secretaria@ctomedicina.com www. ctomedicina.

11

Comentarios de Test a distancia 1. vuelta

Psiquiatra
con antidepresivos. Por eso, es un poco ms frecente en mujeres (el
trastorno bipolar-2 es algo ms frecuente en el sexo femenino).
En ocasiones se detectan alteraciones tiroideas que necesitan ser
corregidas para frenar la ciclacin. En general no responden bien a
la monoterapia con litio; quizs respondan algo mejor al tratamiento
con anticonvulsivos, pero lo ms habitual es utilizar como mnimo dos
estabilizadores. Si algo nos recuerdan estos pacientes es la necesidad de
extremar las precauciones al usar antidepresivos en pacientes bipolares.

es el nivel de desarrollo de la regin (p.ej., mejor en zonas rurales que


en reas urbanas).
Existe una estacionalidad referida al mes de nacimiento de los pacientes,
encontrndose un mayor nmero de pacientes nacidos en los meses
fros del ao, lo que se ha relacionado con diversos factores causales
(infecciones vricas).
Estadsticamente, los pacientes tienen un mayor riesgo de muerte tanto
por causas no naturales (suicidio) como por causas naturales.

Pregunta 40.-R: 2
Como ya dijimos, el litio es el medicamento de primera eleccin en la
prevencin de recadas de la enfermedad bipolar pero no es la solucin
denitiva. Hay pacientes que no toleran sus efectos adversos, mientras
que en otros est contraindicado (insuciencia renal, psoriasis, enfermedad del nodo sinusal). Adems, hay un alto porcentaje de pacientes
(30-40%) que no responden bien a este medicamento:
Podemos predecir que no van a responder al litio:
Las manas atpicas o disfricas (irritables).
Los episodios mixtos (que mezclan sntomas depresivos y manacos).
Los cuadros manacos secundarios a enfermedades mdicas.
Los pacientes con ciclacin rpida.

Pregunta 42.-R: 4
En la actualidad, el diagnstico de esquizofrenia se basa en los siguientes
criterios:
1) Presencia de unos sntomas caractersticos que se han dividido
en positivos (los que predominan en las fases agudas o brotes) y
negativos (que son ms evidentes en las fases residuales).
2) Ausencia de otras enfermedades psiquitricas, enfermedades mdicas, frmacos o txicos que los justiquen.
3) Produccin de un deterioro en el funcionamiento habitual del paciente (acadmico, social, laboral).
4) Una duracin determinada:
- La clasicacin DSM especica que debe ser superior a 6 meses,
comprendiendo los prdromos, el brote psictico (que debe
durar como mnimo un mes, salvo que se trate precozmente)
y la fase residual de recuperacin. Si la duracin es inferior a 6
meses se diagnosticar de trastorno psictico breve si no dura
ms de un mes o de trastorno esquizofreniforme si dura entre
uno y 6 meses.
- La CIE slo acepta el diagnstico de trastorno psictico breve
cuando dura menos de un mes, llamando tambin esquizofrenia
a los trastornos esquizofreniformes de la DSM.

Algunos anticonvulsivos se han convertido en frmacos de referencia


para el trastorno bipolar, sobre todo el cido valproico. Tambin juegan
un papel muy importante en el control de la enfermedad la carbamacepina, su derivado oxcarbacepina o la lamotrigina (este ltimo para las
fases depresivas). Los anticonvulsivos tienen menor toxicidad, lo que
explica su utilizacin en casos mdicamente complejos o pacientes
con intolerancia al litio. Se controlan de una forma similar a su uso en
la epilepsia, aunque no se ha demostrado que los niveles plasmticos
guarden relacin con su ecacia (s con la toxicidad). Otros anticonvulsivos (clonazepam, topiramato, gabapentina, pregabalina) tienen
menor utilidad, aunque pueden usarse como frmacos coadyuvantes
en casos concretos.
Los antipsicticos atpicos han ido demostrando ecacia en distintos
aspectos de la enfermedad. Su accin antimanaca est aceptada con
claridad, existiendo datos sobre la ecacia antidepresiva de algunos de
ellos (quetiapina) y sobre su capacidad de prevenir recadas.

POSITIVOS
Concepto

De novo, no presentes
en la experiencia normal

Prdida de una funcin


psicolgica normal

Sinnimos

Productivos, "psicticos",
"activos"

Decitarios, residuales

De curso breve, agudos


Fciles de identicar
y valorar
Gran acuerdo entre
diferentes entrevistadores
Recuerdan a los de primer
rango de Schneider

Crnicos, estables
en el tiempo
Difciles de valorar
Discrepancias entre
entrevistadores
Recuerdan a los
primarios de Bleuler

Caractersticas

TRASTORNOS PSICTICOS. FRMACOS ANTIPSICTICOS


Pregunta 41.-R: 3
La esquizofrenia es una enfermedad frecuente, afectando a cerca del
1% de la poblacin en algn momento de su vida; teniendo en cuenta
que tan slo 1/3 de los casos se pueden considerar de buen pronstico,
con una recuperacin ms o menos completa, se puede calcular que en
Espaa hay cerca de 300.000 personas con la enfermedad.
Tiende a aparecer en la adolescencia o juventud (de los 15 a los 35 aos),
siendo excepcional su inicio en la infancia y en la vejez.
Afecta por igual a hombres y mujeres, si bien en las mujeres tiene un
inicio algo ms tardo (quizs por el efecto protector de los estrgenos),
lo que determina un mejor pronstico.
No se han descrito variaciones signicativas de su incidencia en funcin de
la clase social, la cultura o la raza; sin embargo, s se encuentran diferencias
en la prevalencia en determinadas regiones del mundo o determinados
grupos poblacionales que pueden explicarse por diferencias en el
pronstico y la mortalidad. As, la mayora de los pacientes tiene un nivel
socioeconmico bajo, como consecuencia del deterioro que produce la
enfermedad (hiptesis del descendimiento social); el pronstico en lo
que se reere a la adaptacin social parece ms favorable cuanto menor
CTO Medicina C/Francisco Silvela, 106 28002 - Madrid

NEGATIVOS

Escalas

Ejemplos

BPRS, PSE, SADS, etc.

PANSS, SANS

Alucinaciones
Delirios
Catatona
Conductas extraas (?)
Disgregacin (?)

Pobreza del lenguaje


Aplanamiento afectivo
Asociabilidad, anhedonia
Dcit de atencin
Afecto inapropiado (?)

Pregunta 42. Sntomas positivos y negativos de la esquizofrenia.


Pregunta 43.-R: 3
Desconocemos cules son las causas precisas de esta enfermedad, pero
se maneja un modelo terico que incluye factores que aumentaran el
riesgo para padecerla (factores predisponentes o de vulnerabilidad)
y otros que contribuiran a su aparicin en individuos predispuestos
(factores precipitantes o desencadenantes).
Dentro de los factores de vulnerabilidad que predisponen a sufrir esquizofrenia destaca, sobre todo, la presencia de antecedentes familiares
de esquizofrenia; la heredabilidad de la esquizofrenia explica el 80%
de la varianza de su aparicin, si bien la penetrancia de la enfermedad

Tfno. (0034) 91 782 43 30/33/34 E-mail: secretaria@ctomedicina.com www. ctomedicina.com

12

Comentarios de Test a distancia 1. vuelta

Psiquiatra
es incompleta y se implican numerosos genes dentro de un modelo
extremadamente complejo.
Otros factores que aumentan en menor grado la posibilidad de sufrir
esquizofrenia son:
Haber nacido en los meses fros del ao (se ha relacionado con
infecciones por el virus de la gripe durante el embarazo).
Antecedentes de problemas obsttricos (el sufrimiento fetal se
asociara con dao cerebral).
Enfermedades graves de la primera infancia (sobre todo las que
afectan directamente al sistema nervioso central, como meningoencefalitis, epilepsia grave, traumatismos craneales severos).
Los factores desencadenantes NO producen esquizofrenia en personas
sin vulnerabilidad; su inuencia es ms evidente en las recadas de la
enfermedad:
Txicos (sobre todo alucingenos, cannabis y estimulantes como la
cocana o las anfetaminas).
Estrs social (especialmente importante para explicar algunas recadas
debidas al abandono del tratamiento).
Enfermedades graves que afectan al sistema nervioso central.
La existencia de claros factores desencadenantes es un dato de mejor
pronstico (siempre ser mejor que el cerebro se rompa como consecuencia de una causa que lo haga espontneamente).
Pregunta 44.-R: 4
Los trastornos del pensamiento son, junto con las alteraciones de la percepcin, los sntomas ms conocidos de la esquizofrenia; los podemos
dividir en trastornos del contenido del pensamiento (ideas delirantes)
y trastornos del curso o la forma del pensamiento.
Dentro de las ideas delirantes, hablamos de ideas delirantes primarias
cuando parecen surgir de forma autnoma, independientes del resto
de fenmenos psicopatolgicos, siendo tpicas de la esquizofrenia y la
paranoia; las ideas delirantes secundarias (o ideas deliroides) proceden
de fenmenos de tipo afectivo (depresin, mana, angustia).
En lo referente a las alteraciones del curso y la forma podemos ver
dicultades para dirigir el pensamiento hacia unas conclusiones lgicas (tangencialidad), apareciendo con frecuencia prdidas de sentido
(descarrilamientos) que acaban por producir la desestructuracin del
pensamiento (disgregacin) que puede recordar la incoherencia de los
pacientes con trastornos cognitivos graves; en otras ocasiones vamos
a encontrar un pensamiento muy empobrecido (alogia), quejndose
los pacientes de interrupciones en curso del pensamiento (bloqueos).
Pregunta 45.-R: 2
Kurt Schneider fue un psiquiatra que se plante facilitar el diagnstico
de la esquizofrenia mediante un estudio de aquellos sntomas cuyo
diagnstico fuera ms fcil, siendo adems sucientemente frecuentes y
razonablemente tpicos. Buscaba lo que hoy en da llamaramos sntomas
de alto valor predictivo positivo, aunque se equivoc al creer que eran
frecuentes (aparecen en slo un tercio de los pacientes), por lo que su
valor predictivo negativo es bajo.
Hay que reconocer que son altamente sugerentes de esquizofrenia,
pero NO patognomnicos. No olvidar que en la esquizofrenia son
mucho ms frecuentes los sntomas negativos, que por desgracia
son mucho ms inespeccos.
Dentro de las alucinaciones destac las auditivas, en concreto aqullas
en las que se escuchan voces humanas que hablan entre s del enfermo
y de su vida (no, por ejemplo, voces que insultan, que son con mucho
las alucinaciones ms frecuentes en toda la psiquiatra).
CTO Medicina C/Francisco Silvela, 106 28002 - Madrid

En los delirios destac tanto su contenido (delirios de control e inuencia frente a los mucho ms frecuentes, pero ms inespeccos delirios
de perjuicio) como la forma de aparicin (percepciones delirantes sin
sentido, frente a las mucho ms frecuentes interpretaciones delirantes,
ms o menos conectadas con la realidad).
Ejemplo de percepcin delirante: veo una persona apoyada en una
farola de la calle, iluminada por su luz; esa luz quiere decir que soy
el nuevo Mesas, y ese es el Demonio que viene a impedir que me
manieste.
Ejemplo de interpretacin delirante: veo una persona apoyada en
una farola de la calle, iluminada por su luz; comienzo a pensar si no
lleva mucho tiempo all y si no estar esperando a que salga para
hacerme dao.
Pregunta 46.-R: 3
En las fases agudas o brotes predominan los sntomas positivos y en las
fases residuales los sntomas negativos. El aplanamiento afectivo es un
sntoma negativo.
Vamos a detenernos un poco en el diagnstico diferencial de los sntomas
negativos; stos pueden deberse a diferentes causas, cada una de ellas
con una solucin especca:
En ocasiones, los sntomas se deben a que el enfermo est deprimido
(es frecuente al principio de la enfermedad, cuando tienen todava
conciencia de su cambio), pudiendo responder al tratamiento antidepresivo.
Por otro lado, los frmacos antipsicticos tienen la capacidad de
producir sntomas extrapiramidales que mimetizan los sntomas
negativos y responden a una reduccin o un cambio de la medicacin.
No hay que olvidar que la repercusin social de la enfermedad (aislamiento, rechazo) conduce a muchos enfermos a una situacin similar
a la descrita en los cuadros decitarios que mejora radicalmente al
comenzar un tratamiento de tipo rehabilitador.
Por ltimo, la persistencia de sntomas positivos intensos puede hacer
que algunos enfermos rechacen el contacto social o se muestren
retrados como consecuencia del miedo que tienen a ser daados
por los dems.
Una vez controlados todos estos sntomas negativos secundarios nos
encontraramos con los sntomas negativos primarios, derivados realmente de la enfermedad.

Pregunta 46. Historia natural de la esquizofrenia.


Pregunta 47.-R: 2
Los factores pronsticos de la esquizofrenia han sido preguntados en
varias ocasiones; podemos dividirlos de una forma sencilla en:
Factores previos al inicio de la enfermedad:
- Sexo (mujeres mejor que varones).
- Antecedentes familiares (mejor si NO hay antecedentes de esquizofrenia).
- Nivel de adaptacin previo (mejor cuanto mayor fuera).
- Personalidad previa (mejor si era normal).
- Nivel de inteligencia previo (mejor si NO era bajo).

Tfno. (0034) 91 782 43 30/33/34 E-mail: secretaria@ctomedicina.com www. ctomedicina.

13

Comentarios de Test a distancia 1. vuelta

Psiquiatra

Factores relacionados con el debut de la enfermedad:


- Edad de inicio (uno de los fundamentales; mejor cuanto ms tarda).
- Presencia de factores precipitantes (mejor si los hay).
- Presencia de sntomas depresivos o manacos (mejor si los hay).
- Presencia de sntomas confusionales (mejor si los hay).
- Inicio rpido (mejor que si el inicio es insidioso).
- Patrn de sntomas (mejor si dominan los positivos).
- Rapidez a la hora de iniciar un tratamiento.
Factores evolutivos:
- Nmero de recadas (fundamentalmente en los 5 o 10 primeros
aos).
- Gravedad de los sntomas negativos (determinantes para el
pronstico social y laboral).
- Resistencia al tratamiento de los sntomas positivos.

Pregunta 48.-R: 3
En la prevencin de las recadas de la enfermedad esquizofrnica resulta
fundamental tanto el control de los factores que favorecen las mismas
(consumo de txicos, manejo del estrs) como el mantenimiento de un
tratamiento antipsictico a largo plazo; sin embargo, la adherencia al
tratamiento farmacolgico en estos pacientes es muy baja y los factores
que inuyen en el abandono del mismo condicionan la probabilidad
de recada:
El perl de efectos secundarios del medicamento: para evitar el
abandono por esta razn debemos individualizar el tratamiento
segn las preferencias del paciente; teniendo en cuenta que los
efectos extrapiramidales son muy mal tolerados, por lo que los
antipsicticos atpicos parecen tener una mejor aceptacin.
La conciencia de enfermedad: puede mejorarse mediante el trabajo
de psicoeducacin en la enfermedad, sobre todo a travs de terapias
grupales.
La gravedad del estrs psicosocial sufrido por el paciente: en muchos
casos es al ambiente familiar la principal fuente de tensin psicolgica; en su da se cre el trmino expresin emocional elevada para
referirse a esas familias en las que se evidenciaban unos mecanismos
patolgicos de adaptacin a la enfermedad; puede disminuirse esta
tensin mediante tcnicas de intervencin familiar, el uso de recursos como centros de da u hospitales de da y, en ltima instancia,
mediante alternativas residenciales.
Pregunta 49.-R: 2
La paranoia o trastorno delirante es una enfermedad sorprendente y poco
frecuente en la que los pacientes tienen bsicamente un nico sntoma
(un delirio), con una llamativa preservacin del resto del funcionamiento
psicolgico y un escaso deterioro de su funcionamiento.
Aunque se pueden ver cuadros paranoicos de inicio agudo, en
respuesta a un estrs identificable (como los descritos en presos
o inmigrantes), la paranoia clsica tiene un inicio insidioso, casi
siempre entrelazado con una personalidad previa peculiar (paranoide, demasiado sensible), de tal forma que cuando el delirio
toma importancia el paciente suele encontrarse en la edad adulta,
sufriendo consecuencias en los terrenos social, familiar o laboral
segn los contenidos del delirio.
Tiene un curso crnico, con una respuesta discreta del delirio al tratamiento; con la medicacin se va a buscar ms un control conductual
que una disolucin del delirio (encapsulacin).
Las clasicaciones actuales aceptan cinco formas de delirios crnicos
paranoicos:
Delirios de persecucin (los ms frecuentes, pero muy poco especcos).
CTO Medicina C/Francisco Silvela, 106 28002 - Madrid

Delirios de celos (celotipia delirante).


Delirios de enfermedad (hipocondra delirante).
Delirios de grandeza.
Delirios de enamoramiento.

Pregunta 50.-R: 3
El mecanismo por el cual los pacientes paranoicos llegan al delirio es
la interpretacin delirante de la realidad; con frecuencia basan sus
delirios en hechos que realmente han sucedido, pero a los que dan
un significado especial que no se aleja de lo razonable (a diferencia
de los delirios esquizofrnicos, cuyos razonamientos pueden ser
increbles y difciles de comprender). Psicolgicamente estos pacientes recurren a la proyeccin como estrategia de afrontamiento
del estrs (mecanismo de defensa), al situar en el exterior la causa
de sus problemas (por ejemplo, si no encuentro algo es porque me
lo han quitado).
Clsicamente no se admite la existencia de alucinaciones en los trastornos delirantes, aunque podemos ver alteraciones de la percepcin
(sobre todo ilusiones tctiles u olfativas) que pudieran confundirse con
actividad alucinatoria.
Pregunta 51.-R: 4
Los antipsicticos clsicos o tpicos ejercen diferentes acciones sobre
los sistemas de neurotransmisin cerebrales:
Poseen la capacidad de bloquear los receptores dopaminrgicos;
el bloqueo D2 se ha relacionado tanto con su efecto antipsictico
como con sus efectos secundarios extrapiramidales (por accin
sobre la va nigroestriada) y endocrinos (hiperprolactinemia por
su accin sobre la va tuberoinfundibular); adems se relaciona
con su capacidad para inhibir el vmito y el hipo inducidos por
quimioterapia.
Los ms antiguos tienen importantes efectos antihistamnicos (que
producen somnolencia y aumento del apetito y del peso), efectos
antiadrenrgicos (con hipotensin ortosttica como consecuencia) y
efectos anticolinrgicos (produciendo el tpico sndrome muscarnico:
sequedad de boca, estreimiento, visin cercana borrosa, retencin
urinaria, alteraciones de memoria).
Pregunta 52.-R: 3
Dentro de los antipsicticos tpicos o neurolpticos encontramos
diferencias en su potencia (capacidad de bloqueo dopaminrgico)
que pueden ser compensadas con un aumento de dosis; usando dosis
equivalentes la ecacia antipsictica es similar en todos ellos, pero al
aumentar la dosis van a aparecer ms efectos sistmicos como consecuencia de los bloqueos colaterales descritos (menor potencia, ms
dosis, mayor sedacin, ms hipotensin, ms acciones anticolinrgicas);
sin embargo, en los antipsicticos menos potentes la presencia de un
bloqueo anticolinrgico signicativo va a disminuir la tasa de efectos
extrapiramidales (puesto que as en la va nigroestriada los dos neurotransmisores estarn bloqueados).
Las consecuencias de la hiperprolactinemia inducida por los antipsicticos son bien conocidas: en mujeres veremos amenorrea, ginecomastia,
galactorrea y disminucin de la libido, pero se han descrito casos de
hipogonadismo grave con repercusin en la masa sea de las pacientes;
en varones, la hiperprolactinemia produce disfunciones sexuales y disminucin de la libido sobre todo. Dentro de los antipsicticos clsicos,
destaca la capacidad del sulpiride para inducir aumentos enormes de
la prolactina, incluso cuando se usa en dosis bajas. Para encontrar un
antipsictico que no modique la prolactina hay que irse al grupo de
antipsicticos atpicos, sobre todo la clozapina y frmacos anes (olanzapina, quetiapina).

Tfno. (0034) 91 782 43 30/33/34 E-mail: secretaria@ctomedicina.com www. ctomedicina.com

14

Comentarios de Test a distancia 1. vuelta

Psiquiatra
Ya existen algunas presentaciones parenterales de los antipsicticos
atpicos, si bien slo la risperidona de accin prolongada (consta) ha
logrado imponerse frente los antipsicticos depot tpicos.
Tienen un precio muy superior a los frmacos tradicionales, sin que
por el momento se haya podido demostrar en estudios de costeefectividad que compensen su precio con una reduccin de otros
costes de la enfermedad.

Pregunta 52. Relacin entre dosis


y potencia en los antipsicticos clsicos.
Pregunta 53.-R: 4
Los modernos antipsicticos atpicos se han convertido en los frmacos
de primera eleccin para el tratamiento de la mayora de los trastornos
psicticos. Se diferencian tericamente de los antipsicticos clsicos en
tres caractersticas:
Producen menos efectos secundarios extrapiramidales y neuroendocrinos (prolactina).
Tienen un mayor efecto antipsictico que los tpicos (tomando como
referencia el haloperidol).
Tienen cierto efecto sobre los sntomas negativos de la enfermedad
(accin de la que carecen los antipsicticos tpicos).
Pregunta 53. Evolucin de los antipsicticos.
La clozapina fue el primer antipsictico atpico y es el que cumple realmente con todas las caractersticas descritas, pero su asociacin con
agranulocitosis ha hecho que pase a ser un frmaco de reserva, utilizado
cuando todos los dems fallan.
Cmo consiguen ese perl farmacolgico diferencial?
Prcticamente todos combinan acciones antidopaminrgicas con
acciones antiserotoninrgicas.
Algunos de ellos tienen la capacidad de bloquear mltiples receptores
(dopaminrgicos, serotoninrgicos, histaminrgicos, adrenrgicos,
colinrgicos), sin que se pueda asegurar en qu bloqueos radica su
ecacia.
Otros deenden el bloqueo dopaminrgico selectivo de las vas
tericamente implicadas en los sntomas de la enfermedad (vas
mesolmbicas y vas mesocorticales), respetando la va nigroestriada,
responsable de los efectos extrapiramidales.
En algunos son las caractersticas del bloqueo dopaminrgico (efecto
de agonista parcial, constante de disociacin del receptor) las que
parecen determinar la diferencia.
Sin embargo, estos frmacos no estn exentos de problemas. La clozapina, la olanzapina y la quetiapina pueden producir un aumento
del apetito y del peso muy llamativos, adems de efectos sobre la
tensin arterial y somnolencia; el aumento de peso se ha relacionado
con alteraciones metablicas (diabetes mellitus tipo 2, dislipemias)
y aumento de la morbimortalidad cardiovascular. La risperidona
produce efectos extrapiramidales a dosis medias y aumenta los
niveles de prolactina incluso en dosis bajas, con las consecuencias
endocrinas conocidas.
CTO Medicina C/Francisco Silvela, 106 28002 - Madrid

Pregunta 54.-R: 1
De todos los efectos secundarios que pueden presentar los antipsicticos,
los ms preguntados son los extrapiramidales; stos van cambiando en
funcin del tiempo:
En las primeras horas o das podemos ver distonas agudas; son
contracciones sostenidas de grupos musculares que pueden resultar dolorosas o peligrosas (distonas larngeas); son especialmente
frecuentes en varones, tanto ms cuanto ms jvenes.
Al cabo de varios das o semanas aparecen dos tipos de efectos
aparentemente opuestos: una reduccin de los movimientos en la
forma de sndrome parkinsoniano o un aumento de la movilidad en
la forma de inquietud (objetiva y subjetiva) o acatisia.
Cuando un paciente ha tomado medicacin antipsictica durante
meses o aos puede presentar movimientos de tipo coreoatetsico
(los ms frecuentes de la musculatura faciobucolingual) denominados
discinesias tardas; en raras ocasiones afectan a msculos del tronco
o las extremidades produciendo una seria limitacin en el paciente;
su respuesta al tratamiento es pobre, pudiendo considerarse casi
irreversibles.
Cada efecto secundario tiene un tratamiento especco:
Anticolinrgicos para las distonas agudas (por va parenteral) y el
parkinsonismo (por va oral): biperideno, trihexifenidilo.
Betabloqueantes (propranolol) o benzodiacepinas (lorazepam) para
la acatisia.
Deplecionantes dopaminrgicos (tetrabenacina) para las discinesias
tardas graves.
Amantadina para el parkinsonismo (no es habitual en Espaa).

Tfno. (0034) 91 782 43 30/33/34 E-mail: secretaria@ctomedicina.com www. ctomedicina.

15

Comentarios de Test a distancia 1. vuelta

Psiquiatra
AGUDOS

SUBAGUDOS
Semanas

TARDOS

Inicio

Das

Aos

Tipo

Distonas

Parkinsonismo

Acatisia

Discinesias

Clnica

Crisis oculgiras
Torticolis

Temblor
Acinesia
Rigidez

Inquietud

Corea facial
Distonas focales
15% graves

Factores de riesgo

Jvenes
Varones
AP tpicos incisivos
Dosis altas

Tratamiento

Anticolinrgicos
parenterales (biperideno)

AP tpicos incisivos
Dosis altas
Dao cerebral
Anticolinrgicos orales
Amantadina

BZD
Betabloqueantes

Ancianos
Mujeres
Dao cerebral
Trastornos afectivos
Anticolinrgicos
Clozapina
Tetrabenacina

Pregunta 54. Efectos extrapiramidales de los antipsicticos.

Un efecto secundario especialmente grave y con componente extrapiramidal es el sndrome neurolptico maligno; en l se combinan:
Elementos extrapiramidales (rigidez extrema).
Hipertermia.
Alteraciones vegetativas (sudoracin, labilidad de la tensin arterial
o la frecuencia cardaca).
Sntomas confusionales.
Tiene una importante mortalidad cuando se deja sin tratamiento, fundamentalmente por las repercusiones renales de la necrosis muscular masiva (por la
rigidez y la hipertermia), siendo bsico su diagnstico precoz (analticamente
veremos leucocitosis, aumento de la CPK y de otras enzimas musculares,
hiperfosfatemia, hiperpotasemia) y su tratamiento inmediato; se suele
indicar el ingreso del paciente en una UCI para un control mdico estricto,
precisando de una serie de medidas fsicas para frenar la hipertermia y de
determinados frmacos (dantroleno [relajante muscular], bromocriptina
[agonista dopaminrgico]) para revertir los sntomas principales.
Pregunta 55.-R: 3
Enlazando con el comentario de la pregunta anterior, tenemos aqu un
paciente que, tras 15 das de tratamiento con un antipsictico, comienza
a presentar inquietud. Esta inquietud inducida por los antipsicticos es
probablemente el sntoma extrapiramidal ms frecuente derivado de
estos frmacos y resulta intensamente desagradable. Recibe el nombre de acatisia (inquietud en griego) y supone una frecuente causa
de abandono del tratamiento. La actitud ante este cuadro consiste en
reducir la dosis del antipsictico y administrar un frmaco corrector (un
betabloqueante o una benzodiacepina), pudindose proceder al cambio
de antipsictico en el caso de que nada funcionara.
Pregunta 56.-R: 4
Cuando hablamos del tratamiento de la enfermedad no debemos
pensar que todo se reduce al uso de frmacos; en ese caso bastara con
asegurarnos el cumplimiento (con formas intramusculares depot) para
controlar la enfermedad.
Tan importante como el tratamiento farmacolgico van a ser todas las
medidas psicosociales destinadas a mejorar la adherencia al tratamiento
(psicoeducacin en la enfermedad, terapias e intervenciones familiares)
y a la rehabilitacin y reinsercin social de los pacientes (tcnicas de
modicacin de conducta, tcnicas de afrontamiento del estrs).
La disponibilidad y gestin de los llamados recursos intermedios
(hospitales de da, centros de da, centros de rehabilitacin psicosocial,
centros de rehabilitacin laboral, recursos residenciales) condicionar
CTO Medicina C/Francisco Silvela, 106 28002 - Madrid

en gran medida el pronstico sociolaboral de la enfermedad. De aqu


que en los ltimos aos se est hablando de nuevos problemas en el
manejo clnico de este tipo de pacientes, como el sndrome de la puerta
giratoria (reingresos repetidos de los mismos pacientes por la ausencia
de recursos externos al hospital) o el problema de los pacientes jvenes
crnicos, especialmente grave en las grandes ciudades. Especial trascendencia tiene la alta prevalencia de consumo de txicos en este grupo
de pacientes (la llamada patologa dual), dado que las sustancias de
abuso tienden a exacerbar los sntomas de su enfermedad y complican
enormemente el tratamiento farmacolgico.
Las terapias dinmicas (derivadas del psicoanlisis) carecen en la actualidad de indicacin en este tipo de pacientes, puesto que precisan de una
capacidad de introspeccin importante y se enfrentan a una enfermedad
con condicionantes neurobiolgicos evidentes.

TRASTORNOS RELACIONADOS CON SUSTANCIAS


Pregunta 57.-R: 5
El alcohol es un depresor del sistema nervioso central y sus efectos dependen de la dosis ingerida, la capacidad de metabolizarlo (tolerancia
farmacocintica) y la adaptacin cerebral al consumo crnico (tolerancia
farmacodinmica). En general, encontramos una buena correlacin entre
la alcoholemia y los efectos clnicos, de tal forma que si una persona
tiene una alcoholemia elevada y no maniesta signos de intoxicacin
podemos asegurar que tiene una dependencia del alcohol.
De forma inversa vemos pacientes que con alcoholemias muy bajas
sufren graves alteraciones conductuales (agresividad, agitacin); son las
llamadas intoxicaciones idiosincrsicas o borracheras patolgicas, y su
origen parece ser la mayor sensibilidad de algunas personas (ancianos,
nios, pacientes con dao cerebral) a las sustancias sedantes (alcohol,
benzodiacepinas, barbitricos). De hecho, algunos alcohlicos crnicos
desarrollan con los aos una tolerancia inversa como consecuencia del
dao cerebral y heptico debido al alcohol.
Cuando se ingiere una cantidad masiva de alcohol puede producirse la
afectacin del tronco del encfalo, apareciendo somnolencia y llegando
en ocasiones a la depresin cardiorrespiratoria. En general, el tratamiento
de las intoxicaciones alcohlicas es puramente sintomtico, evitando las
complicaciones comunes a cualquier coma (neumonas por aspiracin,
sndromes compartimentales por decbito prolongado, hipotermia por
exposicin al aire libre), pero una caracterstica diferencial del coma etlico es la aparicin de hipoglucemia como consecuencia de la inhibicin
de la gluconeognesis y la glucogenlisis; por eso es frecuente que a

Tfno. (0034) 91 782 43 30/33/34 E-mail: secretaria@ctomedicina.com www. ctomedicina.com

16

Comentarios de Test a distancia 1. vuelta

Psiquiatra
estos pacientes se les pauten soluciones glucosadas, con la precaucin
de poner previamente tiamina para no precipitar una encefalopata de
Wernicke; en casos excepcionales se puede recurrir a la dilisis para eliminar cantidades extremas de alcohol y, sobre todo, cuando el paciente
ha ingerido alcoholes no etlicos, mucho ms neurotxicos.

mientras que el segundo no suele poner en riesgo la vida del paciente


y se origina habitualmente a partir de un consumo muy elevado.
Otras diferencias entre ambas entidades pueden observarse en la tabla
siguiente.
DELIRIUM TREMENS

Pregunta 57. Efectos siolgicos de las intoxicaciones


y las abstinencias.
Pregunta 58.-R: 3
Una de las complicaciones ms graves del alcoholismo es el sndrome de
abstinencia severo o delirium tremens. Asocia una importante mortalidad
cuando se deja sin tratamiento, fundamentalmente por las complicaciones hidroelectrolticas secundarias a los vmitos, la sudoracin profusa o
la diarrea, que afectan de una forma especialmente grave a un paciente
que suele tener problemas derivados del alcoholismo en diversos rganos
y sistemas (hepatopata, cardiopata, pancreatitis, etc.).
Para que se produzca un sndrome de abstinencia tiene que existir
tolerancia, y sta se adquiere como consecuencia del consumo crnico
de cantidades elevadas de alcohol; parece que en su fisiopatologa
intervienen mecanismos gabargicos y glutamatrgicos, lo que
quizs explique la elevada incidencia de convulsiones generalizadas
tnico-clnicas.

CDT
Transferrina deciente
en carbohidratos
(desialotransferrina)
GGT
Gamma glutamil
transpeptidasa
VCM
Volumen corpuscular
medio de los hemates
Cociente GOT/GPT
Relacin entre
las transaminasas
Triglicridos, cido rico
Niveles de B12 o flico

Buena sensibilidad y especicidad


Buena rapidez de cambio (2 semanas)
Precio elevado
Buena sensibilidad, baja especicidad
Rapidez de cambio moderada (6 semanas)
Bajo precio
Baja sensibilidad, mejor especicidad
Muy lento en cambiar (120 das)
Bajo precio
Sirve para liar una hepatitis (alcohlica > 2)
No es til si no hay hepatitis
Poco sensibles y poco especcos
Marcadores inespeccos de malnutricin,
no de alcoholismo

Pregunta 58. Marcadores del consumo excesivo de alcohol.


Pregunta 59.-R: 4
En este caso se nos plantea el diagnstico diferencial entre dos cuadros
tpicos del paciente alcohlico crnico: el delirium tremens y la alucinosis
alcohlica. Ambos se caracterizan por la presencia de alucinaciones en un
paciente dependiente del alcohol. Sin embargo, el primero es un cuadro
grave (y potencialmente mortal) desencadenado por la abstinencia,
CTO Medicina C/Francisco Silvela, 106 28002 - Madrid

ALUCINOSIS ALCOHLICA

Desencadenante

Abstinencia brusca

Consumo elevado

Alteracin
de la conciencia

S (delirium)

No (conciencia clara)

Alucinaciones

Visuales (microzoopsias)
Escenogrcas
Inducibles

Auditivas (insultos)

Delirio

Ocupacional

Raro (secundario
a las alucinaciones)

Alteraciones
somticas

Frecuentes

No

Mortalidad

Alta sin tratamiento

Rara

Tratamiento

Asegurar ctes. vitales


BZD, clormetiazol
Evitar NL
Si convulsiones: Mg
Suplementos vitamnicos

Cese del consumo


Haloperidol
Prolaxis de la abstinencia

Tabla 59 Diferencias entre delirium tremens y alucinosis alcohlica.


Pregunta 60.-R: 2
En la prevencin de recadas del alcoholismo tenemos dos enfoques
bien diferentes.
Por un lado, el modelo tradicional se centra en el tratamiento psicolgico con sus diversas variantes (individual, de pareja, familiar,
grupal), en el que son muy conocidos los grupos de autoayuda. Se
propone con frecuencia para estos pacientes el uso de interdictores
(disulram, cianamida), frmacos que bloquean el metabolismo del
alcohol en uno de sus pasos intermedios (aldehdo deshidrogenasa),
provocando la acumulacin de acetaldehdo, el cual desencadena
una reaccin histaminrgica. La base del tratamiento es conductual;
se trata de disminuir una conducta problemtica mediante el miedo
a una consecuencia negativa (castigo); por tanto, es necesario que el
paciente est al tanto de su tratamiento, siendo necesario que rme
un consentimiento informado.
En los ltimos aos el concepto de craving ha ido ganando fuerza en el
tratamiento de las drogodependencias. El tipo ms habitual de craving
es el deseo que aparece condicionado a estmulos ambientales que
se relacionan con el consumo; se propuso el uso de acamprosato (un
modulador gabargico y glutamatrgico) para este problema pero su
ecacia es muy discutible.
Por otro lado, algunos pacientes cuentan una llamativa prdida de
control tras el consumo de una pequea cantidad del txico; en estos
casos la naltrexona (antagonista opioide) ofrece una ecacia clnicamente
signicativa, siendo el frmaco ms utilizado en el tratamiento a largo
plazo del alcoholismo.
Pregunta 61.-R: 4
En la dependencia de la herona, el tratamiento farmacolgico es ms
sencillo al existir frmacos especcos para el sistema opioide endgeno,
tanto agonistas como antagonistas.
En la intoxicacin el exceso de actividad de los opiceos produce una
disminucin del nivel de conciencia, que asocia depresin respiratoria
en los casos graves; la naloxona es el antagonista utilizado en estos
casos por va parenteral; su corta vida media (menos de 4 horas) hace
necesaria la administracin consecutiva de varias dosis para revertir por
completo los efectos del txico.

Tfno. (0034) 91 782 43 30/33/34 E-mail: secretaria@ctomedicina.com www. ctomedicina.

17

Comentarios de Test a distancia 1. vuelta

Psiquiatra
Durante la abstinencia se aprecian sntomas de dcit de actividad de
los opiceos (mialgias, diarrea) y sntomas derivados de la hiperactividad
adrenrgica (HTA, taquicardia, midriasis, temblor). Se puede tratar con
frmacos opiceos (metadona, codena) o con frmacos que reduzca la
actividad adrenrgica (clonidina, guanfacina).

INTOXICACIN

S. ABSTINENCIA /
DESINTOXICACIN

Causas

Sobredosis accidental
Intento de suicidio

Abandono del consumo


Administracin de sustancias
antagonistas, agonistas/
antagonistas o agonistas
parciales

Clnica

MIOSIS
Depresin
cardiorrespiratoria
Alteracin del nivel
de conciencia

MIDRIASIS
Abandono de hiperactividad
adrenrgica (diarrea,
rinorrea...)
Deseo de consumir

NALOXONA i.v.

2 opciones:
a) Sustitutivo: metadona en la
dosis equivalente al consumo
y reduccin gradual en 5-19
das (de ELECCIN)
b) Sintomtico: disminuir
hiperactividad simptica
Agonistas -2 (clonidina)

Tratamiento

Dado que la accin fundamental de esta sustancia es catecolaminrgica


(dopaminrgica, sobre todo) muchos de los frmacos utilizados actan
en estos sistemas.
El uso de antidepresivos (tricclicos, inhibidores selectivos, etc.) se basa
en que aumentan el tono catecolaminrgico y alivian la sintomatologa
depresiva asociada al cese del consumo. Dado que el consumo crnico
obliga a una adaptacin de los sistemas dopaminrgicos cerebrales se
propone el uso de diversos frmacos dopaminrgicos (desde la L-dopa
a los agonistas ms modernos) para aliviar algunos sntomas (disforia,
craving). Finalmente se plantea la utilidad de algunos anticonvulsivos a
la hora de reducir el deseo por consumir.
BENZODIACEPINAS de entrada)
EXCESO DE NA/A:
Vasoconstriccin
crisis HTA hemorragias
isquemia cardaca... arritmias
isquemia cerebral... epilepsia
EXCESO DE DA/A:
Delirios, alucinaciones
Agitacin

CONTROL EN UCI segn


complicaciones)
Monitorizacin ECG y TA
NO usar betabloqueantes (riesgo
de agravamiento paradjico)
Antipsicticos?
Riesgo de convulsiones

Pregunta 63. Tratamiento de la intoxicacin por cocana.

Pregunta 61. Intoxicacin y desintoxicacin de opiceos.


Pregunta 62.-R: 2
La abstinencia de la herona es un cuadro clnico aparatoso pero sin
gravedad; por tanto, y a diferencia de la abstinencia del alcohol, NO es
una urgencia mdica; los pacientes no deben recibir tratamiento de
forma puntual en un servicio de urgencias, salvo que deban quedar
ingresados por otro motivo; la actitud ms correcta es derivarles a centros especializados para que inicien un tratamiento reglado destinado
no slo a la abstinencia sino a la prevencin de recadas.
Para favorecer el mantenimiento de la abstinencia se han probado
diferentes frmacos, aunque existen programas de tratamiento en los
que no se utiliza ayuda farmacolgica alguna.
La naltrexona (antagonista opioide) puede bloquear los efectos de una
dosis puntual de herona, evitando el refuerzo positivo del consumo;
sin embargo, este bloqueo es competitivo y puede ser revertido con
una dosis sucientemente alta de herona, por lo que el paciente
debe comprometerse a no intentar vencerlo (se le pedir que d su
consentimiento informado), pues se provocara una intoxicacin grave
al haber perdido la tolerancia; por este motivo solamente se propone
para pacientes altamente motivados, sin caractersticas impulsivas y sin
problemas mdicos o psiquitricos asociados.
La metadona (agonista opioide) consigue disminuir de forma ms
ecaz las complicaciones legales y mdicas del consumo de herona,
permitiendo la progresiva normalizacin del paciente en todas las
esferas afectadas por la drogodependencia; a dosis elevadas parece
tener adems un efecto anticraving. Se plantea como un tratamiento a
largo plazo, pudiendo necesitarse aos hasta que se pueda plantear su
retirada; es el tratamiento recomendado para los pacientes refractarios
a otros tratamientos, los heroinmanos con problemas psiquitricos o
mdicos graves, las embarazadas y los politoxicmanos.
Pregunta 63.-R: 3
En el tratamiento de la dependencia de la cocana, las opciones de
apoyo farmacolgico son mucho ms limitadas y de hecho ningn medicamento ha demostrado una ecacia signicativa a la hora de reducir
el riesgo de recada.
CTO Medicina C/Francisco Silvela, 106 28002 - Madrid

TRASTORNOS COGNOSCITIVOS: DELIRIUM, DEMENCIAS


Y AMNESIAS
Pregunta 64.-R: 4
Este paciente sufre un delirium o sndrome confusional agudo; esta
alteracin del estado mental es el cuadro psiquitrico ms frecuente en
enfermos ingresados en hospitales generales, como consecuencia de
los procesos mdicos, las intervenciones quirrgicas o los tratamientos
farmacolgicos; el sustrato siopatolgico es una encefalopata aguda
cuyo origen pueden ser problemas originados en el cerebro o fuera de l
(procesos metablicos, txicos, frmacos, etc.). El sntoma inicial del delirium
es una alteracin de la atencin y la concentracin, que con frecuencia
conducen a la desorientacin (primero en el tiempo, ms adelante en
el espacio); es frecuente asistir a grandes uctuaciones de la clnica, en
ocasiones relacionadas con factores externos (p.ej., empeoran al llegar la
noche, pues la menor iluminacin hace que interpreten peor el entorno).
Pregunta 65.-R: 4
Se describen tres formas de delirium (agitado, estuporoso y mixto) en
funcin del patrn de comportamiento. En realidad, en un mismo enfermo
o en un mismo proceso patolgico lo ms frecuente es la forma mixta.
Hay dos aspectos clnicamente importantes del delirium. En primer lugar,
la AGITACIN de algunos delirium puede suponer un riesgo para la salud
del paciente y de las personas que le rodean; por eso llaman al psiquiatra,
como experto en sedar enfermos agitados, para elegir el frmaco que
controle el comportamiento agresivo, sin empeorar el estado fsico.
El haloperidol es el frmaco de eleccin para el control de cualquier cuadro de agitacin, pues carece de efectos cardiovasculares y respiratorios;
su margen de seguridad es muy elevado, y con esas dosis curiosamente
no se producen los temidos efectos extrapiramidales de estos frmacos.
Los antipsicticos ms sedantes (clorpromacina, levomepromacina, zuclopentixol) tienen el riesgo de producir una hipotensin grave y tienen
importantes efectos anticolinrgicos que agravaran la confusin mental.
Los antipsicticos atpicos se estn usando cada vez ms para estos cuadros
de agitacin, si bien, cuando se hace precisa la va intramuscular (lo que
no es nada raro) el haloperidol se convierte en casi la nica alternativa.

Tfno. (0034) 91 782 43 30/33/34 E-mail: secretaria@ctomedicina.com www. ctomedicina.com

18

Comentarios de Test a distancia 1. vuelta

Psiquiatra

Pregunta 65. Caractersticas clnicas del delirium (sndrome confusional agudo).


Las benzodiacepinas a dosis elevadas pueden empeorar el estado mental
y producir depresin respiratoria, sobre todo si el paciente est tomando
otros frmacos con efectos sedantes, sin embargo, son el tratamiento
de eleccin en los sndromes de abstinencia alcohlica grave (delirium
tremens), pues corrigen de forma especca la siopatologa del sndrome.
Pregunta 66.-R: 3
No hay que olvidar que todo delirium tiene una CAUSA que conviene
aclarar cuanto antes, pues implica una morbimortalidad notable; por
eso realizamos diferentes pruebas para diagnosticar el origen del mismo (anlisis de sangre [hemograma, bioqumica completa, serologas,
hemocultivos, etc.] y de orina, ECG, EEG, pruebas de imagen [Rx de
trax, TC], puncin lumbar). El test de Rorscharch (test de las manchas
de tinta) es una prueba psicolgica proyectiva que explora los aspectos
inconscientes de la personalidad; es totalmente absurdo aplicarlo a estos
pacientes, con una alteracin grave de su nivel de conciencia.

cia REM y en la segunda se acorta con frecuencia; sin embargo, es una


prueba compleja y con niveles de sensibilidad y especicidad bajos, por
lo que no se suele utilizar.
Pregunta 68.-R: 3

DEMENCIA
Curso
Inicio
Progresin
Duracin
H personal
H familiar

CTO Medicina C/Francisco Silvela, 106 28002 - Madrid

Insidioso

Agudo

Lenta

Rpida

Larga

Breve

Normal

Depresin

Demencia

Tr. afectivos

Clnica
Actitud ante la enfermedad

Pregunta 67.-R: 4
Cuando la combinacin de fallos de memoria, desorientacin, disnomia
y dispraxias, y la demostracin de alteraciones en la exploracin neuropsicolgica elemental (MEC o Minimental) hace que sospechemos una
posible demencia, debemos seguir un sistema de diagnstico diferencial
para descartar posibles enfermedades que tienen un carcter reversible
o, al menos, algo ms benigno que las demencias degenerativas.
El primer paso es descartar que no exista una causa externa que est
inuyendo en el funcionamiento cerebral hasta el punto de producir
una disfuncin cognitiva grave: intoxicaciones crnicas con frmacos
sedantes, abuso de txicos, alteraciones metablicas (insuciencia
renal, hipotiroidismo, encefalopata heptica), anemias graves, lesiones
ocupantes de espacio intracraneal (tumores, quistes, abscesos), enfermedades infecciosas o inamatorias con tratamiento ecaz (slis, esclerosis
mltiple), dcit vitamnicos, etc.
Por tanto, al paciente con sospecha de demencia deben realizrsele pruebas analticas (hemograma, bioqumica plasmtica completa, serologas
[slis, VIH], sistemtico de orina, estudio funcional tiroideo, niveles de
B12 y flico), ECG y pruebas de imagen (Rx trax, TC), para descartar las
principales causas de demencia reversibles o tratables.
La polisomnografa, sin embargo, slo puede ayudar a diferenciar la
demencia de la depresin, pues en la primera puede aumentar la laten-

PSEUDODEMENCIA
DEPRESIVA

Ocultacin

Exageracin

Congruencia entre conducta


y deterioro

No

Fluctuaciones clnicas

No

Empeoramiento nocturno

Frecuente

Raro

Humor

Depresivo

Lbil, indiferente

Adecuado

Prdida precoz

Inters social

Exploracin
Atencin y concentracin

Afectadas

Conservadas

Respuestas tpicas

Aproximadas

"No s"

Dcit de memoria

Reciente > remota

Reciente=remota

Lagunas mnsicas

Raras

Frecuentes

Esfuerzo en los test

No

Congruente

Variable

Respuesta a la privacin
de sueo

Empeoramiento

Mejora

Respuesta al amobarbital

Confusin

Cogniciones
depresivas

Respuesta en los test

Pregunta 68. Diferencias entre demencia


y depresin.

Tfno. (0034) 91 782 43 30/33/34 E-mail: secretaria@ctomedicina.com www. ctomedicina.

19

Comentarios de Test a distancia 1. vuelta

Psiquiatra
OTROS TRASTORNOS MENTALES: PERSONALIDAD,
ALIMENTACIN, SUEO, INFANTILES
Pregunta 69.-R: 4
El trastorno lmite de la personalidad es probablemente el que ms complicaciones psiquitricas produce, el que ms medicacin va a recibir, el
que ms veces va a acudir a Urgencias, el que ms veces va a ingresar y
el que mayor riesgo de suicidio tiene.
En l se mezclan sntomas de todas las lneas de la psicopatologa
(ansiosos, emocionales, psicticos, alimentacin, impulsividad, abuso
de sustancias, etc.), en general de una intensidad importante pero de
una duracin breve, y, casi siempre, en respuesta a alguna circunstancia
vivida como frustrante. Probablemente por sto se trate del trastorno de
la personalidad que ms puede responder al tratamiento farmacolgico
(antidepresivos, antipsicticos, estabilizadores), teniendo el cuidado de
elegir frmacos con un elevado nivel de seguridad en el caso de intoxicacin (no es raro que estos pacientes protagonicen autolesiones de
carcter impulsivo). Se ha relacionado este trastorno con el antecedente
de abusos sexuales en la infancia, si bien este dato es muy cuestionado.
Pregunta 70.-R: 1
Los trastornos de personalidad aumentan el riesgo de presentar trastornos mentales, existiendo algunas relaciones especialmente claras:
Los esquizotpicos sufren con frecuencia episodios psicticos breves;
en ellos encontramos una elevada frecuencia de antecedentes familiares de esquizofrenia; en algunas clasicaciones se considera a la
esquizotipia una forma latente de esquizofrenia pues sus sntomas
recuerdan en ocasiones a los de esta enfermedad.
Los esquizoides tambin se relacionan con las enfermedades psicticas, siendo la personalidad anormal que con ms frecuencia se
relaciona con la aparicin de esquizofrenia; de hecho, sus rasgos
se solapan con los sntomas negativos de la esquizofrenia en sus
formas simple o residual (pero a diferencia de esos pacientes, los
esquizoides siempre han sido as).
Los paranoides se relacionan muy fuertemente con los trastornos delirantes, ya sean agudos (psicosis reactivas breves) o crnicos (paranoia).
Pregunta 71.-R: 3

Pregunta 71. Esquema de los trastornos de la conducta alimentaria.


CTO Medicina C/Francisco Silvela, 106 28002 - Madrid

En los trastornos alimentarios fundamentales (anorexia y bulimia nerviosa) el sntoma central es el mismo: una preocupacin PATOLGICA
por la posibilidad de engordar. Este miedo condiciona su relacin con
la comida, a la que ven ms como una enemiga que como un medio
necesario para conseguir una actividad normal. A qu se debe ese
miedo? Muchas veces no llegamos a saberlo, pero subyacen una serie
de pensamientos distorsionados y sobrevalorados (pero NO delirantes)
acerca de las consecuencias del aumento de peso en sus vidas. No es raro
encontrar adems distorsiones en la percepcin de su esquema corporal,
pero algunas pacientes, reconociendo que estn delgadas, no soportan la
idea de recuperar peso o no son capaces de normalizar su alimentacin.
Cuando este trastorno aparece en una chica con una alta capacidad de
autocontrol (generalmente con rasgos obsesivos de personalidad) veremos cmo va a ser capaz de seguir una dieta restrictiva de forma estricta,
aguantando su deseo de comer (la palabra anorexia sugiere la falta de
apetito, lo cual no es cierto hasta que no se alcanza la emaciacin); como
consecuencia de esa dieta se produce una notable prdida de peso (se
considera importante una prdida del 15% del peso esperable para su
edad y talla o un IMC inferior a 17,5); la desnutricin acarrea numerosas
complicaciones fsicas, entre las que destaca la amenorrea. Es lo que
denominamos anorexia nerviosa.
Cuando el trastorno afecta a chicas con un menor autocontrol (con rasgos
ms impulsivos) la dieta va a verse interrumpida por atracones bulmicos
en los que se ingiere una enorme cantidad de alimento en un tiempo
llamativamente corto, con una sensacin de prdida de control; para
compensar los atracones aparecen una serie de conductas destinadas
a favorecer la prdida de peso, que van desde las conductas purgantes
(vmitos autoprovocados, abuso de laxantes, uso de diurticos) a otras
menos agresivas (alternancia de ayuno y atracones, ejercicio compulsivo).
Lo llamaremos bulimia nerviosa.
Sin embargo, vemos tambin algunas formas mixtas; lo ms frecuente es
que con la cronicacin de una anorexia nerviosa las pacientes comiencen
a presentar conductas purgantes SIN tener atracones, para no absorber
lo poco que coman (anorexia restrictiva-purgativa). Adems, con el paso
de los aos, las pacientes con anorexia llegan a perder el control sobre su
alimentacin y comienzan a presentar atracones (lo que algunos llaman
bulimarexia). Ms raro es que una chica con bulimia llegue a enganchar
un perodo sucientemente largo de autocontrol como para que pierda
peso de forma signicativa.
Pregunta 72.-R: 4
Los atracones bulmicos guardan una estrecha relacin con una baja
capacidad de autocontrol y los rasgos impulsivos de personalidad; por
eso no es raro encontrar en las pacientes bulmicas una mayor frecuencia
de otros comportamientos impulsivos (abuso de drogas, autolesiones,
sntomas depresivos, promiscuidad, etc.), asocindose algunos casos al
trastorno lmite de la personalidad.
Suelen ser ms frecuentes por la noche (cuando es menos probable que
sean descubiertas) y tras un perodo de ayuno. Los alimentos deben ser
fciles de tragar y no es raro que se seleccionen sobre todo dulces (la urgencia del atracn no hace posible la seleccin de alimentos hipocalricos).
De todas las conductas alimentarias anormales, slo los atracones son susceptibles de cierto tratamiento farmacolgico; la medicacin de eleccin
son los ISRS, utilizndose dosis mucho ms altas que las habituales para
una depresin. El topiramato se ha puesto de moda como alternativa a
los ISRS para regular la impulsividad relacionada con la comida.
En general, el tratamiento de las pacientes con trastornos de la conducta
alimentaria es multidisciplinar, implicndose especialistas en psiquiatra,
en endocrinologa y nutricin (el bajo peso es la principal causa de ingreso), en modicacin de conducta (para ayudar en el autocontrol de
los atracones y los vmitos) y en psicoterapia (para explorar los motivos

Tfno. (0034) 91 782 43 30/33/34 E-mail: secretaria@ctomedicina.com www. ctomedicina.com

20

Comentarios de Test a distancia 1. vuelta

Psiquiatra
SUEO
VIGILIA ACTIVA
(OJOS ABIERTOS)

RELAJACIN
(OJOS CERRADOS)

No REM 75% (SUEO SINCRONIZADO)


Fase 1 (5%)

EEG

Beta (>12 Hz)


frontal

EMG

Activo

EOG

Movimientos
rpidos

Fase 2 (45%)

Theta (4-8 Hz)

Alfa
(8-12 Hz) occipital

Complejos K husos del sueo

Descenso

< 50%

> 50%

Disminuido

Media

Beta (>12 Hz)


ondas en dientes
de sierra
Atona

Ausentes

Supercial

FC, TA, F.resp

Fase 4 (13%)

Sueo "delta" (<4Hz)

Movimientos lentos rotatorios


en balancn

Profundidad

Fase 3 (12%)

REM 25%
(SUEO
DESINCRONIZADO)

Sueo "profundo"

Rpidos, conjugados
Media
Inestabilidad,
arritmias, apneas

Descenso, estabilidad
PRL (+)

Regulacin
hormonal

GH (+), TSH (-), ACTH (-)

Fenmenos
fsicos
T

Homeotermia

Parasomnias

Poiquilotermia
Bruxismo

Sonambulismo, terror nocturno

Pesadilla

Pregunta 73. Fases del sueo y fenmenos siolgicos y patolgicos asociados.


que subyacen a esa preocupacin por la alimentacin).
Pregunta 73.-R: 4
Vase la tabla correspondiente a esta pregunta.
Pregunta 74.-R: 4
La narcolepsia (sndrome de Gelineau) es una enfermedad poco frecuente
en la que se combinan:
Ataques incoercibles del sueo (tpicamente en sueo REM).
Con la aparicin de caractersticas del sueo REM en la vigilia:
- Cataplejia (prdida brusca del tono muscular ante las emociones
bruscas).
- Parlisis del sueo (falta de recuperacin del tono muscular tras
el despertar).
- Alucinaciones hipnaggicas e hipnopmpicas (aparicin o persistencia de las imgenes de los sueos en la fase de transicin
entre vigilia y sueo).
En su tratamiento se combinan una serie de medidas higinicas que buscan
respetar los perodos naturales de sueo e interrumpir la actividad, con
siestas programadas, para disminuir la frecuencia de ataques de sueo con
diversos medicamentos destinados a aliviar la somnolencia (estimulantes
anfetamnicos, modanil) o los fenmenos REM (antidepresivos).
Tiene un fuerte componente hereditario, asocindose al HLA-DQB1 en
un porcentaje muy alto de los casos.
Pregunta 75.-R: 2
El autismo infantil debuta clnicamente antes de los 3 aos de edad,
siendo posible detectar manifestaciones al poco tiempo del nacimiento.
Lo fundamental es un dcit en la interaccin social, con ausencia de
reciprocidad social o emocional. Es tambin destacable la alteracin de
la comunicacin, con retraso en el desarrollo del lenguaje, dicultad para
mantener una conversacin, as como el uso estereotipado del mismo o
bien un lenguaje idiosincrsico. La comunicacin no verbal tambin est
reducida. La conducta es repetitiva, sin juego creativo. Son frecuentes
las estereotipias y los manierismos, manifestando resistencia a cualquier
cambio del entorno, incluso llegando a episodios de agitacin. Se asocia
CTO Medicina C/Francisco Silvela, 106 28002 - Madrid

con retraso mental de grado moderado en el 75% de los casos y con


convulsiones en el 25%.
Trastornos anes al autismo son el sndrome de Asperger (una especia
de autismo benigno en el que se conserva el lenguaje y no hay retraso
mental) y el trastorno desintegrativo (una forma de autismo tardo que
debuta tras dos aos de desarrollo normal).
El sndrome de Rett es un trastorno generalizado del desarrollo que
no forma parte del espectro autista; afecta mayoritariamente a nias y
aparece tras varios meses de desarrollo psicomotor normal; junto con las
graves alteraciones psicopatolgicas se evidencia una desaceleracin del
crecimiento ceflico y otros signos neurolgicos; se produce en el 95%
de los casos por una mutacin de novo (no hereditaria) en el gen MECP2,
situado en el brazo largo del cromosoma X.
Pregunta 76.-R: 4
El uso de psicofrmacos en los trastornos mentales de la infancia y la
adolescencia est insuencientemente estudiado, siendo muy pocos los
ensayos clnicos realizados en gran medida por la controversia hacia el uso
de frmacos psiquitricos en este grupo de edad, dado que las hiptesis
dominantes atribuyen la mayora de los trastornos mentales de la infancia y la adolescencia a conictos psicolgicos o problemas relacionales.
Sin embargo, en algunas entidades est claramente establecida la utilidad
de la medicacin, sin que por ello se descarten los abordajes psicolgicos
o sociales:
Trastorno por dcit de atencin con hiperactividad:
- De primera eleccin: estimulantes anfetamnicos (metilfenidato
en Espaa).
- De segunda eleccin: antidepresivos (tricclicos, atomoxetina);
modanilo.
Trastorno de La Tourette:
- De primera eleccin: antipsicticos tpicos incisivos (pimocida,
haloperidol).
- De segunda eleccin: antipsicticos atpicos, clonidina.
Enuresis nocturna:
- De primera eleccin: tcnicas de modicacin de conducta (sistemas de alarma).
- De segunda eleccin: desmopresina, imipramina.

Tfno. (0034) 91 782 43 30/33/34 E-mail: secretaria@ctomedicina.com www. ctomedicina.

21

Comentarios de Test a distancia 1. vuelta

Psiquiatra
Pregunta 77.-R: 4
El trastorno por dcit de atencin es probablemente el diagnstico
psiquitrico ms frecuente en la edad infantil. El sntoma fundamental
es la dicultad para sostener la atencin un tiempo prolongado, lo que
provoca que los nios sean fcilmente distrables por cualquier estmulo y
se muestren sumamente inquietos. Suele debutar en la primera infancia,
pero se hace ms evidente al iniciar la escolarizacin. De sus tres esferas
sintomticas (dcit de atencin, hiperactividad, impulsividad), las dos
primeras suelen mejorar rpidamente con el tratamiento farmacolgico,
que siempre debe combinarse con una intervencin psicolgica en el
medio escolar y con la familia; los estimulantes anfetamnicos mejoran la
capacidad atencional y disminuyen la distraibilidad, permitiendo que el
nio aproveche las horas escolares y se reduzcan los comportamientos
problemticos. Cuando no se diagnostica a tiempo o predomina la im-

CTO Medicina C/Francisco Silvela, 106 28002 - Madrid

pulsividad es frecuente que vayan apareciendo conductas antisociales


(robos, mentiras, peleas), con una clara tendencia al mantenimiento
de esos comportamientos y al diagnstico de trastorno antisocial de
la personalidad en la edad adulta. Un porcentaje importante de los pacientes seguirn presentando sntomas en la edad adulta, siendo muy
elevada su comorbilidad con otros trastornos psiquitricos, sobre todo
los relacionados con el abuso de sustancias.
Conviene tener en cuenta este diagnstico cuando nos enfrentamos
a un nio con problemas de comportamiento en el medio escolar, si
bien es necesario que se demuestre la existencia de sntomas de las dos
esferas principales (dcit de atencin, hiperactividad), pues existen
otras muchas causas de mal comportamiento en un nio (trastorno
negativista-desaante, depresin con predominio de la irritabilidad,
problemas de adaptacin en pacientes con discapacidad intelectual).

Tfno. (0034) 91 782 43 30/33/34 E-mail: secretaria@ctomedicina.com www. ctomedicina.com

22

S-ar putea să vă placă și